You are on page 1of 109

TEMPLATE CASE BASED SCENARIO EXAMINATION (CBSE) 20/21

Kamus Dewan Bahasa :


Melakukan sesuatu dengan bersungguh-sungguh (biasanya dengan tujuan mengejar waktu)

UiTM PULUN
TEMPLATE CASE BASED SCENARIO EXAMINATION (CBSE) 20/21

SURGERY
BIL APPROACH TOPIC PAGE
1 Achalasia 1
Approach to dysphagia
2 Esophageal CA 6

3 Variceal bleeding 10

4 Approach to UGIB Non-variceal bleed: PUD 16

5 Non-variceal bleed: Gastric CA 22

6 Painless bleeding: Colorectal CA 28


Approach to LGIB
7 Painful bleeding 34

8 Peripheral vascular disease (PVD) 38


Approach to vascular
9 Peripheral arterial disease (PAD) 45

10 Approach to inguinal swelling Inguinal hernia 51

11 Approach to breast lump Breast carcinoma 58

12 Approach to neck swelling Thyroid carcinoma 64

13 Acute pancreatitis 70
Approach to abdominal pain
14 Hepatocellular carcinoma 78

15 Approach to jaundice Obstructive jaundice 87

16 Approach to hematuria Urolithiasis 98

17 Approach to LUTS BPH, prostate carcinoma 103

UiTM PULUN
TEMPLATE CASE BASED SCENARIO EXAMINATION (CBSE) 20/21

ACHALASIA

CASE SCENARIO
Name Mr Aman
Age 33 years old
Race, Gender Malay, male
Underlying
Chief complaint Dysphagia
Duration 3 months
Associated symptoms (if any) + duration

Achalasia • Early age unless pseudoachalasia (>60 y.o)


• Progressive to fluid
• Immediate regurgitation of food
• Halitosis
Esophageal carcinoma • Progressive solid food
• Risk factor: barret esophagus, GERD, family hx of esophageal CA
Erosion • Dysphagia
• Hematemesis
GERD • Long-standing gastriris, acid reflux
• RF: lying down after meal
Inflammatory bowel disease • Particular CD; a/w other systemic manifestation
• a/w bloody diarrhoea
Scleroderma • Elderly
• Thickening of skin, furrowing of mouth, CREST syndrome

UiTM 1
TEMPLATE CASE BASED SCENARIO EXAMINATION (CBSE) 20/21

PART 1

HISTORY
MUST GET GOOD TO GET
EXPLORE CHIEF COMPLAINT COMPLICATIONS FROM THE DISEASE

Dysphagia  age of onset Symptoms • Cough


S Site where patient feel food stuck of aspiration • Chocking
(neck, middle chest, lower chest) pneumonia • fever
O Progressive/ sudden/ Intermittent • SOB (night-time)
Difficulty initiate swallowing/ feel food stuck Symptoms • Bone: bone pain, fracture
of (osteoporosis),
C Painful/ painless
metastasis hypercalcemia sx
Dysphagia to solid/ liquid/ both • Lung: dyspnea, pleuritic
R - chest pain, haemoptysis
A  Vomiting/ regurgitation undigested food • Liver: jaundice, RUQ pain,
 Choking/ coughing/ nasal regurgitation ascites
• Brain: seizure, headache,
 Retrosternal chest pain/ hoarseness of
drowsy (increase ICP)
voice
Locally • Hoarseness
 Regurgitation of old food & halitosis advanced • Thoracic spine pain
T After meal? After specific exposure? In the tumor (thoracic spine spread)
early morning? • Fever, cough (trachea-
E - esophageal fistula)
Tumor • Hematemesis
S - bleeding • Malena
Relieves Repeated swallowing  achalasia • History of chronic used of
NSAIDs
ASSOCIATED SYMPTOMS
Reflux  dyspepsia, heartburn RISK FACTORS
symptoms  acid brash/ water-brush,
 postural aggravation on lying down Squamous (SCC) AdenoCA
Constitutional  LOA /significant LOW  Age >60 y/o
symptoms  Fever  Gender: male
 Anorexia/ lethargy  Family hx of esophageal CA
Malnutrition  dehydration  Alcohol + smoking  Chronic GERD
 weight loss  High ingestion of  Barret
Anaemic  Fatigue nitrosamine esophagus
symptoms  Breathlessness  Achalasia  Obesity
 Palpitation  Nutritional deficiency:  Smoking
 Dyspnoea on exertion lack vegetable + fruits
 Fainting/ Light headedness
 Headache
 Weakness
Systemic  Weakness (stroke)
symptom  Proximal myopathy
 CREST syndrome
(Calcinosis, Raynaud phenomenon,
esophageal dysmotility,
Sclerodactyly, telangiectasia) .

UiTM 2
TEMPLATE CASE BASED SCENARIO EXAMINATION (CBSE) 20/21

PHYSICAL EXAMINATION
MUST GET GOOD TO GET
General symptoms: Peritonism sign
 Body built  cachexia  Abdominal tenderness/ guarding
 Total Parenteral Nutrition (TPN)  Visible peristalsis
 Feeding: NG tube, gastrotomy/ jejunostomy  Absent bowel sound

Vital sign: BP, PR, RR, temperature  esp BP, PR, temp:
hypotension/ tachycardia/ slight rise Sign of  Cervical
metastasis lymphadenopathy
Sign of  Sunken eyes  Left supraclavicular node
dehydration  Dry mucous membrane (Virchow’s node)
 Loss skin turgor  Hepatomegaly
 Ascites
Sign of anaemia  Conjunctival pallor/ jaundice
Sign of  Febrile
(when metastatic to liver)
aspiration  Lung crepitation
 Palmar crease pallor
pneumonia  Decreased air entry right
Neck  Laryngeal pouch lower lobe (right
examination  Goiter bronchial more vertical)
Abdominal  Cough impulse Neurological  Cranial nerve
examination  Mass of abdomen/ ascites/ examination examination TRO
previous scar neuromuscular dz
Per Rectal  Malaena Hand grip test  Normal
examination
Iron deficiency  Angular stomatitis
anaemia (IDA)  Glossitis
 Koilonychia

PART 2

FULL DIAGNOSIS
Presentation Achalasia
Secondary to
With underlying (if any)

UiTM 3
TEMPLATE CASE BASED SCENARIO EXAMINATION (CBSE) 20/21

INVESTIGATION
MUST GET GOOD TO GET
DIAGNOSTIC TO ASSESS COMPLICATION

1. Oesophageal manometry: Laboratory


 Lack of progressive peristalsis (aperistalsis) Full blood  TRO anaemia
 Abnormally high pressures at the LES count  Raised WCC in aspiration
 Incomplete LES relaxation on swallowing
(FBC) pneumonia
2. Barium swallow :  Hct raised if dehydrated
 “Bird’s beak” tapering stricture of distal Renal  TRO electrolyte imbalance
oesophagus with proximal dilatation profile d/t vomiting/ poor oral
intake
3. Oesophagogastroduodenoscopy (OGDS): for  Raise urea and creatinine
diagnostic + therapeutic purpose (d/t dehydration)
 TRO mechanical stricture (at gastroesophageal
Arterial  TRO exclude metabolic
junction (GEJ)
 TRO esophagitis d/t stasis of food blood alkalosis and respiratory
gases acidosis
(ABG)
Imaging Imaging
Chest X-ray  TRO aspiration pneumonia Endoscopic  Beneficial TRO
(consolidation) ultrasound pseudoachalasia
 widened mediastinum  Finding: difficulty advancing
 air-fluid level (large amounts endoscope through the GEJ
of retained food of a non-dilated
 accumulate fluid in dilated oesophagus
oesophagus  Locoregional staging for
 absence of gastric air bubble) esophageal CA
CT TAP If suspected malignancy
.

UiTM 4
TEMPLATE CASE BASED SCENARIO EXAMINATION (CBSE) 20/21

MANAGEMENT
MUST GET GOOD TO GET
DEFINITIVE Endoscopy management
 Per-oral Endoscopic Myotomy
Surgery Laparoscopic Heller esophagomyotomy (POEM)
 Pneumatic balloon dilatation (65%
Principle: (myotomy of lower esophagus) + anterior 180
of patients improve, 40% response
partial (Dor) fundoplication  to reduce post-operative rate at 5 years)
 Risks: perforation with dilatation,
reflux
recurrence of symptoms

SUPPORTIVE

Lifestyle & Medical (aim: decrease LES tone)


 CCB (nifedipine) and nitrates (nitroglycerine)
 Endoscopic injection of botulinum toxin (block Ach release
from nerve terminals)

Nutritional support:
1. Enteral feeding
2. Parenteral feeding
Advantages Disadvantages
 Beneficial for  Costly
long-term use  Central venous line
 Useful for pts complication
with large calorie  Risk of pneumothorax
and nutrients  Sepsis
needs  Metabolic complication

UiTM 5
TEMPLATE CASE BASED SCENARIO EXAMINATION (CBSE) 20/21

OESOPHAGEAL CANCER

CASE SCENARIO
Chief complaint Mrs S, a 76 years old women, retired greengrocer presents to her GP because she is
having difficulty swallowing for the past 3 weeks.

PART 1

HISTORY
MUST GET GOOD TO GET
EXPLORE THE CHIEF COMPLAINT COMPLICATIONS
 Duration of dysphagia  Symptoms of malnutrition:
 Progressive or intermittent  anaemic sx (SOB, dizziness/ palpitation/ pre-
 Dysphagia to solids/fluids/both syncopal attack
 Painless or painful  dehydration
 Site where it feel stucked  weight loss

ASSOCIATED SYMPTOMS  Symptoms of aspiration pneumonia  cough/


 Any coughing, choking, gurgling, dysphonia, fever/ SOB
vomiting, haematemesis, melaena
 Heartburn, waterbrash  Tumor bleeding  malaena/ haematemesis
 Neurological symptoms – difficulty
coordinating swallowing, slow eating, extra  Symptoms of metastasis (haematogenous
effort to eat/chew, tiredness after eating, spread)
early dysphagia for liquids  Bone: bone pain, fracture (osteoporosis),
 Rheumato symptoms – CREST for systemic hypercalcemia sx
sclerosis  Lung: dyspnea, pleuritic chest pain,
 Symptoms of metastases haemoptysis
 PMH – GERD, peptic ulcer, multiple sclerosis,  Liver: jaundice, RUQ pain, ascites
parkinson disease  Brain: seizure, headache, drowsy (increase
 Drug history – CCB, nitrates (relax smooth ms ICP)
and cause reflux by decreasing esophagel
tone); NSAIDS, steroids, biphosphonates RISK FACTORS
(peptic ulcer) Squamous (SCC) AdenoCA
 Age >60 y/o
 Gender: male
 Family hx of esophageal CA
 Alcohol + smoking  Chronic GERD
 High ingestion of  Barret
nitrosamine esophagus
 Achalasia  Obesity
 Nutritional deficiency:  Smoking
lack vegetable + fruits

UiTM 6
TEMPLATE CASE BASED SCENARIO EXAMINATION (CBSE) 20/21

PHYSICAL EXAMINATION
MUST GET GOOD TO GET
 Assessment of nutritional status (cachexia, signs of
anaemia, signs of hypoalbuminaemia, BMI)
 Left supraclavicular LN
 Cranial nerve pathology (bulbar palsy)
 Signs of GI malignancy
o Cachetic
o Palpable virchow’s node
o If extend to cardia of stomach, may be
palpable in thin patient
 Neck mass – large pharyngeal pouch, goiter
 Features of CREST syndrome
 Koilonychia – IDA secondary to plummer vinson
syndrome
 Anthropometry
 Hand dynamometry – evaluate strength of hands
 PEFR

DIFFERENTIAL DIAGNOSIS
Achalasia 
Esophageal 
carcinoma
Erosion 
GERD 

PART 2

CASE SUMMARY
Mrs S, a 76 y/o had progressive difficulty swallowing solid food over the past 3 weeks and it feels like the food
getting stuck midway down her throat. However, she has no difficulty with actual chewing. She had no
coughing, choking, dysphonia or gurgling after eating or drinking. She has LOA and LOW. Previous hx of GERD.
O/E, she is cachetic and there is palpable lymph node at left supraclavicular area

FULL DIAGNOSIS
Presentation Dysphagia
Secondary to Esophageal Cancer
With underlying (if any)

UiTM 7
TEMPLATE CASE BASED SCENARIO EXAMINATION (CBSE) 20/21

INVESTIGATION
MUST GET GOOD TO GET
DIAGNOSTIC TO ASSESS COMPLICATION
 FBC – anaemia, raised white cell count
1. Barium swallow (non-invasive) - any lesion, (aspiration pneumonia)
cancer (mid oesophageal stricture)  Renal profile – raised urea, creatinine
(dehydration)
2. Oesophagogastroduodenoscopy (OGDS) and  LFT – low albumin
biopsy or brush cytology
STAGING (IF ANY)
3. Oesophageal manometry: 1. CT TAP : if suspected malignancy + for staging
 Motility disorder (checking the pressure in 2. Endoscopic ultrasound:
LES and peristaltic wave  Locoregional staging of tumor staging
 Beneficial for T,N staging
3. Chest-Xray
 TRO aspiration pneumonia
 widened mediastinum + air-fluid level (large
amounts of retained food + fluid in dilated
oesophagus with absence of gastric air
bubble)
4. Surgical Laparoscopic Staging (most beneficial for
patients with adenocarcinoma)

Specific pre-operative IX:


1. Renal profile: TRO electrolyte disturbance from
vomiting/ poor oral intake
2. Lung function test
3. Echocardiogram

UiTM 8
TEMPLATE CASE BASED SCENARIO EXAMINATION (CBSE) 20/21

MANAGEMENT
MUST GET GOOD TO GET
DEFINITIVE MANAGEMENT POST ESOPHAGECTOMY
Surgical
 Oesophagectomy (cx : breakdown of the 1. Analgesia – IV or epidural (i.e. morphine or
anastomosis, pneumonia, cardiac bupivacaine)
arrhythmia)
i. Transthoracic Esophagectomy (TTE) : Ivor Lewis 2. Gastrointestinal
TTE or McKeown Esophagectomy i. NBM – 5 to 7 days
ii. Trans-hiatal Esophagectomy (THE) ii. Jejunostomy feeding tube – placed
during surgery and start feeding on POD
Neo/adjuvant therapy 2-3
 Radiotherapy iii. NG tube on low level intermittent /
 Chemotherapy - 5-Fluorouracil and continuous suction
Cisplatin iv. Gastrografin swallow – day 5 to 7 to
check for anastomotic leak before
Metal stent initiating oral intake
v. Escalation of feeds as tolerated – aim
Ablation with laser/ argon beam plasma for 6 to 8 small frequent meals each day
coagulation
3. Prevention of complications
Endoscopal mucosal reaction i. Early mobilization
ii. Aggressive Pulmonary Rehabilitation
SUPPORTIVE (i.e. incentive spirometry, deep
Nutrition delivery breathing exercise etc.)
 Parenteral/enteral

UiTM 9
TEMPLATE CASE BASED SCENARIO EXAMINATION (CBSE) 20/21

UGIB: VARICEAL BLEED

CASE SCENARIO
Name Ali
Age 55
Race, Gender
Underlying
Chief complaint Vomit out blood
Duration
Associated symptoms (if any) + duration For 3 days

PART 1

HISTORY
MUST GET GOOD TO GET
EXPLORE THE CHIEF COMPLAINT ETIOLOGY/RISK FACTOR
Risk factors • IV drug use - sharing needle
 Vomiting for viral • Tattoos
 Frequency hepatitis • Acupuncture
 Amount • Sexual history (unprotected sex,
 Projectile/ non projectile multiple partner - men to men sex)
 Content (Billious/Blood/Food) • Travel hx
 Colour • Hx of blood transfusion
 Previous hx of vomiting • Family hx
Alcohol • Quantity and duration
 Melena intake • Has ascites diminished/resolved
 Anaemic Symptom intermittently in relation to
abstinence from alcohol?
ASSOCIATED SYMPTOMS • CAGE
 Lethargy Drug • Hepatotoxic drug
 Nausea and/or vomiting (traditional/herbal medication)
 Pruritus Family • Wilson’s disease (high copper)
 Weight gain history • Hemochromatosis (high iron)
 Shortness of breath Malignancy • Prior history of cancer
 Early satiety • Associated abdominal pain
• History of long-standing cirrhosis
Coagulopathy Bleeding tendency: followed by abrupt development of
(reducing clotting - Easily bruising ascites: consider hepatocellular
factor synthesis) - Gum bleeding carcinoma
- Epistaxis • Loss of weight/Loss of appetite
Jaundice Sign of • Consider spontaneous bacterial
(impaired breakdown of bilirubin) infection peritonitis (SBP) in patient with
Encephalopathy - Drowsiness ascites and fever or abdominal
(poor detoxification of - Inversed sleep pattern pain/ discomfort
harmful substances) - Confusion
- Inappropriate behavior. ROS
Ascites (poor albumin - Abdominal distension Urinary sx – Oliguria  hepatorenal syndrome
synthesis + increased - Abdominal discomfort Bowel sx – Constipation  hypertensive encephalopathy
portal pressure due to - Leg edema
scarring)  varices)

UiTM 10
TEMPLATE CASE BASED SCENARIO EXAMINATION (CBSE) 20/21

PMH
 Any underlying chronic liver disease

DIFFERENTIAL DIAGNOSIS
Peptic/duodenal  History of dyspepsia, previous H. pylori infections, previous OGDS done
ulcer  Drug History – NSAIDs, antiplatelets, steroids, anticoagulants, TCM
Oesophageal  Painful UGIB
erosions
Malignancy  Epigastric pain  Early satiety
 Dyspepsia  Dysphagia
 Anemia  Nausea/vomiting,
 Melena  Bloatedness
 Hematemesis  LOW/LOA
Mallory-Weiss  The history should include forceful retching preceding any bleeding
tear

UiTM 11
TEMPLATE CASE BASED SCENARIO EXAMINATION (CBSE) 20/21

PHYSICAL EXAMINATION
MUST GET GOOD TO GET
GENERAL MAIN CLINICAL CONSEQUENCES OF PORTAL
1. Oriented to time, place and person HYPERTENSION
2. Vital sign : BP, RR, PR, temperature 1. Ascites
(Hemodynamic status) 2. Formation of portosystemic shunt
3. Body built (cachexic) 3. Splenomegaly

HAND 1. Clubbing - Per rectal : Melena


2. Leuconychia Reduced serum Leuconychia
3. Palmar erythema albumin Ascites
4. Flapping tremor Impaired  Palmar erythema
5. Dupuytren contracture estrogen  Loss of secondary sexual
ARM 6. Loss of axillary hair metabolism characteristic (axillary and
7. Bruising pubic hair loss, testicular
8. Needle mark atrophy)
9. Scratch mark  Gynaecomastia
10. Tatoo  Spider naevi
EYE 11. Conjunctival pallor
12. Sclera jaundice Portal Splenomegaly
MOUTH 13. Glossitis hypertension Caput medusa
NECK, FACE, 14. Lymphadenopathy Hepatic Not oriented to TPP
CHEST 15. Gynaecomastia encephalopathy Flapping tremor
16. Spider naevi
ABDOMINAL 17. Caput medusa
18. Abdominal distension
(ascites) - umbilicus (normal,
flat, everted)
19. Shrunken liver
20. Hepatospenomegaly
INGUINAL 21. Hernia (ascites)
22. Testicular atrophy
LEG 23. Bilateral pitting edema
.

PART 2

FULL DIAGNOSIS
Presentation Upper Gastrointestinal Bleeding
Secondary to Acute Variceal Bleed
With underlying (if any) Chronic Hepatitis

UiTM 12
TEMPLATE CASE BASED SCENARIO EXAMINATION (CBSE) 20/21

INVESTIGATION
MUST GET GOOD TO GET
DIAGNOSTIC Ascites fluid examination following peritoneal tap:

LABORATORY
1. FBC (anaemia, leucocytosis,
thrombocytopaenia)
2. RP ( ↑ creatinine, urea) - hepatorenal
syndrome
3. GXM.GSH
4. LFT( ↑ AST, ALT, bilirubin)
5. Coag Profile (Prolong PT, INR >1.5) - affect
Vitamin K dependent clotting factor. 1972
6. Serum glucose, serum lactate - impaired
gluconeogenesis - hypoglycemia, cell starve-
activate anaerobic metabolism
7. Viral serology for Hepatitis A, B, C
8. Biochemical test ( a-antitrypsin, alpha-
fetoprotein) - HCC
9. Autoimmune marker (ANA, IgG) - autoimmune
hepatits

IMAGING
1. Emergency OGDS
2. Abdominal ultrasound with doppler
Hepatosplenomegaly, hepatic vessel
thrombosis, hepatic surface nodularity.
3. CT abdomen and MRI
Useful in diagnosing hepatic malignancies

UiTM 13
TEMPLATE CASE BASED SCENARIO EXAMINATION (CBSE) 20/21

MANAGEMENT
MUST GET GOOD TO GET
DEFINITIVE CONSERVATIVE (Lifestyle, diet etc)
Resuscitate (manage in critical care area)
To stop the bleeding  Maintain airway – KIV intubation If patient has
altered mental state (encephalopathy) or
hematemesis is copious
 Breathing - supplemental high flow oxygen, maintain
Sp02 >94%
 Establish 2 or more large bore peripheral IV lines
 Monitoring - Vitals, ECG, pulse oximeter, urine
output (IDC)
 Labs  GXM (4units), FBC, U/E/Cr, PT/PTT, ±LFT,
±Cardiac Enzymes
 Infuse fluids - 1 litre N/S fast and reassess
parameters
 ICU bed and facilities should be made available

Pharmacology:
1. IV broad-spectrum antibiotic 7 days - (ciprofloxacin
500mg bd or ceftriaxone 1g / day)
2. IV somatostatin (250ug bolus followed by 250ug/h
infusion for 3-5days) or IV octreotide (50mcg bolus
1. Resuscitation followed by 50mcg/hour for 3-5days) @
2. ICU admission : correction of volume depletion, 3. ± IV Terlipressin (2mg Q6H) (synthetic vasopressin) is
the vasoactive drug of choice with a 34% mortality
maintain airway
relative risk reduction (CI in patients with IHD)
3. Blood transfusion to maintain at 8g/dL. FFP and 4. IV omeprazole 80mg bolus
platelets to be transfused if there is 5. IV Vitamin K (10mg) – should be given routinely to
coagulopathy/thrombocytopenia cirrhotic with coagulopathy
4. Reduce portal pressure by decreasing splanchnic 6. ± Recombinant activated factor VII (rFVIIa) – for
blood flow correcting PT in cirrhotic
 IV infusion Vasopressin
Ballon
 IV Nitroglycerin
Sengstaken-Blakemore tube / Minnesota tube
 Terlipressin (analogue of vasopressin with Maximum 24 hours – temporary deflate after 12 hours to
lesser side effect) prevent pressure necrosis) in patients with uncontrollable
 Octreotide bleeding for whom a more definitive therapy is planned
5. Ballon temponade : Senstaken Blakemore (i.e. TIPS or endoscopic therapy)
6. TIPS : Transjugular Intahepatic portal shunt
7. Prophylaxis antibiotic : Ceftriaxone
Definitive Management (endoscopy and TIPSS)
8. Protein restriction is adovacated bcos there is
risk of hepatic encephalopathy Endoscopy
9. Avoid hypoglycemia 1. Sclerotherapy
2. Variceal band ligation
Bleeding Prophylaxis
• Non-selective beta-blockers (Propranolol &
Nadolol)
• Omeprazole

UiTM 14
TEMPLATE CASE BASED SCENARIO EXAMINATION (CBSE) 20/21

TIPSS

UiTM 15
TEMPLATE CASE BASED SCENARIO EXAMINATION (CBSE) 20/21

UGIB: NON VARICEAL BLEEDING : PUD

CASE SCENARIO
Name Madam E
Age 44 years old
Race, Gender Indian lady
Underlying
Chief complaint Vomiting of blood
Duration On the day of admission
Associated symptoms (if any) + duration

Peptic ulcer disease (gastric • Coffee-ground vomitus


ulcer @ duodenal ulcer)
• Lifestyle: smoking, alcohol, stress
• Medication: NSAIDs, chemotherapy, Aspirin used
• Helicobacter pylori infection
• Have pain associated with food intake
Oesophageal varices • Fresh blood hematemesis
• Projectile vomiting
• May/ may not positive history for high risk behaviour  portal HPT
Gastric CA • Early satiety
• Metastatic sx
• Constitutional symptom
• High consumption of nitrates, salty food
• Smokers, pylori infection
• Family history of gastric CA, gastritis
Erosion (esophageal, gastric, • Dull/ burning pain/ frequent burping/ LOA/ bloatedness
duodenal)
• Cause: direct trauma (NG tube), radiation, NSAIDs, alcohol, stress (major
trauma, surgery, severe illness
Oesophagitis, gastritis, • Odynophagia/ dysphagia/ feeding intolerance/ heartburn/ chest pain
duodenitis
• Perforation/ hemorrhage/ stricture
• Acute: History of corrosive material/ alcohol
• Chronic: acid reflux (GERD), chemical, alcohol, smoking, radiation

UiTM 16
TEMPLATE CASE BASED SCENARIO EXAMINATION (CBSE) 20/21

PART 1

HISTORY
MUST GET GOOD TO GET
EXPLORE CHIEF COMPLAINT COMPLICATIONS FROM THE DISEASE

Hematemesis Complication Block : IO/ gastric-outlet obstruction:


Nature Malaena/ hematochezia/ haemoptysis in luminal constipation/ vomiting abdominal
S organ distension/ pain
O Sudden/ progressive/ constant Burrow: fistula between adjacent
organ pancreatitis, cholangitis
C Painful/ painless
Burst: peritonitis; sudden/acute onset
Projectile/ non-projectile diffuse @ localized abdominal pain,
R May radiate to back Metastatic • Bone: bone pain, fracture
A Indigestion/ dyspepsia/ dysphagia symptoms (osteoporosis), hypercalcemia sx
Epigastric pain (esp gastric • Lung: dyspnea, pleuritic chest
CA) pain, haemoptysis
T -
• Liver: jaundice, RUQ pain, ascites
E Exacerbates with food • Brain: seizure, headache, drowsy
S - (increase ICP)
Colour Fresh blood/ coffee-ground vomitus
CLD disease • Ascites
Amount Scanty/ profuse/ a/w blood clot RF • Increase somnolence/ aggressive
• Abdominal pain
ASSOCIATED SYMPTOMS • Jaundice
Anaemic • Fatigue
symptoms • Breathlessness RISK FACTORS
• Palpitation PUD Older age; Male> female
• Dyspnoea on exertion  Helicobacter pylori infection
• Fainting/ Light headedness  Lifestyle: smoking, alcohol, stress
• Headache  Medication: NSAIDs, Aspirin
Constitutional • LOA chemotherapy used
symptoms • Significant LOW
• Fever Esophageal  Multiple partner
• Early satiety varices  Chronic Alcohol, IVDU
• Anorexia, lethargy  History of blood transfusion
 Family history/ personal history of
Past medical • Chronic smoker
Hepatitis B and C
history • Chronic hepatitis infection
 Drug-induced hepatotoxicity:
Gastric Outlet Vomiting, dehydration,
anti-TB (isoniazid), PCM, NSAIDs
Obstruction aspiration +/- pneumonia  Previous regular banding / OGDS
(GOO)  Vomiting directly after feeding screening + banding in CLD patient
 Fever, SOB, chocking Gastric CA  high nitrate, salty food
 smokers, H.pylori infection,
 family hx of gastric CA, gastritis,
 radiation exposure
Mallory-  hx of retching/ frequent vomiting/
weiss tear coughing/ hiccup
 hyperemesis gravidarum
.

UiTM 17
TEMPLATE CASE BASED SCENARIO EXAMINATION (CBSE) 20/21

PHYSICAL EXAMINATION
MUST GET GOOD TO GET
General appearance: If comes with peritonitis;
 Pallor/ conjunctival pallor Tender and Guarding on palpation
 Cachexic
GOO (gastric CA): succussion splash
Vital sign: BP, PR, RR, temperature  esp BP,PR (hypovolemic shock
 hypotension, tachycardia) Peritonism sign

BMI  Cachexic  Abdominal tenderness/ guarding


 BMI  Rebound tenderness
 Visible peristalsis
Sign of anaemia Face  conjunctival pallor
 Absent bowel sound
 pallor of mucus
membrane
Metastatic sign:
Cardiac short systolic flow murmur
Auscultation at aortic area
Pulse  tachycardia Sign of  Cervical
 bounding pulse metastasis lymphadenopathy
 collapsing pulse  Left supraclavicular
node (Virchow’s node)
Hands pallor of palmar creases
 Hepatomegaly
Abdominal  any evidence of stigmata chronic liver  Ascites
examination disease?  bone tenderness
 Jaundice  pleural effusion
Non- variceal  Bleeding from mouth/ nose
cause sign  Palpable surpraclavicular node
(Virchow node)  gastric CA

Sign of  Ascites
hypoalbuminemia  Pleural effusion
 Pedal edema
 Leukonychia

Per rectal  Malaenic stool


examination
.

PART 2

FULL DIAGNOSIS
Presentation Peptic Ulcer Disease
Secondary to
With underlying (if any)

UiTM 18
TEMPLATE CASE BASED SCENARIO EXAMINATION (CBSE) 20/21

INVESTIGATION
MUST GET GOOD TO GET
DIAGNOSTIC TO ASSESS COMPLICATION

1) Oesophagogastroduodenoscopy (OGDS): for Laboratory


diagnostic + therapeutic purpose  Forrest Full blood  TRO anaemia
Classification count  Raised WCC in aspiration
 Locate the ulcer  peptic/ duodenal ulcer
(FBC) pneumonia
 Diagnostic  Biopsy to rule out malignancy @
rapid urease testing (CLO test) TRO H.pylori Renal  TRO electrolyte imbalance d/t
infection (yellow  red (positive) profile vomiting/ poor oral intake
 Therapeutic  Ligation of bleeding vessels  Raise urea and creatinine (d/t
 Prognostication of bleeding risk dehydration)
Imaging
Erect chest To see evidence air under diaphragm
X-ray (pneumoperitoneum)  perforated
viscus

2) Barium swallow/ meal :


 it is less accurate at defining mucosal disease or
distinguishing benign/ malignant ulcer disease.

3) Urea Breath Test: screen for H. Pylori Infection

Laboratory
Fasting serum screen for gastrinoma (Zollinger-
gastrin Ellison Syndrome)

Serum Calcium screen for multiple endocrine


neoplasia (MEN)
Stool Antigen screen for H. Pylori Infection
Test
Serologic screen for H. Pylori Infection (not
Testing (IgG and recommended)
IgA ELISA tests):
.

UiTM 19
TEMPLATE CASE BASED SCENARIO EXAMINATION (CBSE) 20/21

MANAGEMENT
MUST GET GOOD TO GET
DEFINITVE: Surgical
• Need for surgical intervention is declining with
Eradication therapy of H.pylori (1st line: triple @ quadruple
widespread use of H2 receptor antagonist
therapy)
(famotidine) and proton pump inhibitors
 Document eradication by endoscopy + CLO test, urea (omeprazole)
breath test or stool serology
• Indication:
1st line: triple PPI: (for 6 weeks),  refractory haemorrhage
therapy omeprazole 20mg BD  perforation
 gastric outlet obstruction  major
(1 PPI, 2 Abx) Antibiotics: for 10-14 days indication for surgical indications
 amoxicillin 1g BD,
 clarithromycin 500mg BD Gastric Billroth gastrectomy
ulcer • Wedge Excision
In penicillin-allergic patients, substitute • Antrectomy with inclusion of
amoxicillin with metronidazole 400mg BD
ulcer, depending on ulcer
2nd line Treatment failure occurs in up to 20% 
(quadruple location
treat with quadruple therapy: for 7-14 day
therapy) • Total Gastrectomy
 Colloidal bismuth sub-citrate 120mg Duodenal 1) Truncal vagotomy with
QDS, ulcer Pyloroplasty
 tetracycline 500mg QDS, 2) Truncal vagotomy with
 metronidazole 400mg BD, antrectomy and Billroth 1 /2
 omeprazole 20mg BD 3) Highly selective vagotomy
(HSV)

Endoscopic Therapy ± follow up Endoscopic Therapy


a) Therapeutic Endoscopy (see above / below)
b) Follow-up Endoscopy
• Re-scope in 6 weeks to document gastric ulcer
healing  If ulcer still present, biopsy ulcer again
(exclude malignancy for GU) and also do antral biopsy
for rapid urease test – CLO test (to confirm eradication
of H. pylori)
• Non-healing gastric ulcers  resected surgically

Billroth II

UiTM 20
TEMPLATE CASE BASED SCENARIO EXAMINATION (CBSE) 20/21

UiTM 21
TEMPLATE CASE BASED SCENARIO EXAMINATION (CBSE) 20/21

UGIB: NON VARICEAL BLEEDING : GASTRIC CARCINOMA

CASE SCENARIO
Name Madam E
Age 44 years old
Race, Gender Indian lady
Family hx of gastric carcinoma
Underlying
Chief complaint Vomiting of blood
Duration On the day of admission
Associated symptoms (if any) + duration

Peptic ulcer disease (gastric • Coffee-ground vomitus


ulcer @ duodenal ulcer)
• Lifestyle: smoking, alcohol, stress
• Medication: NSAIDs, chemotherapy, Aspirin used
• Helicobacter pylori infection
• Have pain associated with food intake
Oesophageal varices • Fresh blood hematemesis
• Projectile vomiting
• May/ may not positive history for high risk behaviour
Gastric CA • Early satiety
• Metastatic sx
• Constitutional symptom
• High consumption of nitrates, salty food
• Smokers, pylori infection
• Family history of gastric CA, gastritis
Erosion (esophageal, gastric, • Dull/ burning pain/ frequent burping/ LOA/ bloatedness
duodenal)
• Cause: direct trauma (NG tube), radiation, NSAIDs, alcohol, stress (major
trauma, surgery, severe illness
Oesophagitis, gastritis, • Odynophagia/ dysphagia/ feeding intolerance/ heartburn/ chest pain 1
duodenitis
• Perforation/ hemorrhage/ stricture
• Acute: History of corrosive material/ alcohol
• Chronic: acid reflux (GERD), chemical, alcohol, smoking, radiation

UiTM 22
TEMPLATE CASE BASED SCENARIO EXAMINATION (CBSE) 20/21

PART 1

HISTORY
MUST GET GOOD TO GET
EXPLORE CHIEF COMPLAINT COMPLICATIONS FROM THE DISEASE

Hematemesis Complication Block : IO/ gastric-outlet obstruction:


Nature Malaena/ hematochezia/ haemoptysis in luminal constipation/ abdominal distension/
S organ vomiting/ abdominal pain
O Sudden/ progressive/ constant Burrow: fistula between adjacent
C Painful/ painless organ pancreatitis, cholangitis
Projectile/ non-projectile Burst: peritonitis; sudden/acute onset
R May radiate to back diffuse @ localized abdominal pain,
A Epigastric pain Metastatic • Bone: bone pain, fracture
Nausea/ vomiting symptoms (osteoporosis), hypercalcemia sx
Early satiety (esp gastric • Lung: dyspnea, pleuritic chest
CA)
Indigestion/ dyspepsia/ dysphagia pain, haemoptysis
T - • Liver: jaundice, RUQ pain, ascites
E Exacerbates with food • Brain: seizure, headache, drowsy
S - (increase ICP)
Colour Fresh blood/ coffee-ground vomitus
Amount Scanty/ profuse/ a/w blood clot RISK FACTORS
Gastric CA  high nitrate, salty food
 smokers, H.pylori infection,
ASSOCIATED SYMPTOMS  family hx of gastric CA, gastritis,
Anaemic • Fatigue  radiation exposure
symptoms • Breathlessness PUD Older age; Male> female
• Palpitation  Helicobacter pylori infection
 Lifestyle: smoking, alcohol, stress
• Dyspnoea on exertion
 Medication: NSAIDs, Aspirin
• Fainting/ Light headedness
chemotherapy used
• Headache 
Constitutional • LOA Esophageal  Multiple partner
symptoms • Significant LOW varices  Chronic Alcohol, IVDU
• Fever  History of blood transfusion
• Early satiety  Family history/ personal history of
• Anorexia, lethargy Hepatitis B and C
Past medical • Chronic smoker  Drug-induced hepatotoxicity:
anti-TB (isoniazid), PCM, NSAIDs
history • Chronic hepatitis infection
 Previous regular banding / OGDS
Bleeding • Hematochezia screening + banding in CLD patient
tumor • Malaena Mallory-  hx of retching/ frequent vomiting/
Gastric Outlet Vomiting, dehydration, weiss tear coughing/ hiccup
Obstruction aspiration +/- pneumonia  hyperemesis gravidarum
(GOO)  Vomiting directly after .
feeding
 Fever, SOB
.

UiTM 23
TEMPLATE CASE BASED SCENARIO EXAMINATION (CBSE) 20/21

PHYSICAL EXAMINATION
MUST GET GOOD TO GET
General appearance: If comes with peritonitis;
 Pallor/ conjunctival pallor Tender and Guarding on palpation
 Cachexic
GOO (gastric CA): succussion splash
Vital sign: BP, PR, RR, temperature  esp BP,PR (hypovolemic shock
 hypotension, tachycardia) Peritonism sign

BMI  Cachexic  Abdominal tenderness/ guarding


 BMI  Rebound tenderness
 Visible peristalsis
Sign of anaemia Face  conjunctival pallor
 Absent bowel sound
 pallor of mucus
membrane
Cardiac short systolic flow murmur Sign of  Cervical
Auscultation at aortic area metastasis lymphadenopathy
Pulse  tachycardia  Left supraclavicular
 bounding pulse node (Virchow’s node)
 collapsing pulse  Hepatomegaly
Hands pallor of palmar creases  Ascites
 bone tenderness
Abdominal  any evidence of stigmata chronic liver
 pleural effusion
examination disease?
 Jaundice
Non- variceal  Bleeding from mouth/ nose
cause sign  Palpable surpraclavicular node
(Virchow node)  gastric CA

Sign of  Ascites
hypoalbuminemia  Pleural effusion
 Pedal edema
 Leukonychia

Per rectal  Malaenic stool


examination
.

PART 2

FULL DIAGNOSIS
Presentation Gastric CA
Secondary to
With underlying (if any)

UiTM 24
TEMPLATE CASE BASED SCENARIO EXAMINATION (CBSE) 20/21

INVESTIGATION
MUST GET GOOD TO GET
DIAGNOSTIC TO ASSESS COMPLICATION

1. Oesophagogastroduodenoscopy (OGDS): for Laboratory


diagnostic + therapeutic purpose  Full blood  TRO anaemia
 Direct visualization and multiple biopsies of count  Raised WCC in aspiration
suspicious lesion (biopsy edge of ulcer)
(FBC) pneumonia
 Diagnostic  Biopsy to rule out malignancy @
rapid urease testing (CLO test) TRO H.pylori Renal  TRO electrolyte imbalance
infection (yellow  red (positive) profile d/t vomiting/ poor oral
 Therapeutic  Ligation of bleeding vessels intake
 Raise urea and creatinine
2. Barium swallow/ meal : (d/t dehydration) a/w
 it is less accurate at defining mucosal disease or
gastric outlet obstruction
distinguishing benign/ malignant ulcer disease.
Liver  TRO liver metastasis
STAGING function  Synthetic function of liver
test (LFT)  Albumin as nutritional
Laboratory marker
Endoscopic Locoregional staging (T, N stage) Imaging
ultrasound • Superior to CT in delineating Erect chest To see evidence air under
depth of tumour invasion (gold diaphragm
X-ray
standard for T staging) (pneumoperitoneum) 
• identifying perigastric perforated viscus
lymphadenopathy (N staging
accuracy = 70%)
CT scan CT TAP: Assess for haematogenous Pre-op assessment
spread to distant organ (i.e. most ECG/ chest X-ray
commonly liver mets), detect mets Investigation for
GXM/ coagulation baseline
dz  in form of malignant ascites profile

CT abdomen : gastric cardia tumor


PET scan  Combines spatial resolution of
CT with contrast resolution of
PET
 Superior to CT scan alone
.

UiTM 25
TEMPLATE CASE BASED SCENARIO EXAMINATION (CBSE) 20/21

MANAGEMENT
MUST GET GOOD TO GET
DEFINITVE: CONSERVATIVE (Lifestyle, diet etc)

Based on staging of gastric CA  Dumping syndromes*


1. Surgery Early dumping syndrome: food enters small bowel
2. Chemo/radiotherapy too rapidly, drawing fluid into bowel by osmosis
(i.e. rapid fluid shift)  treat by eating small
Endoscopic Therapy  Excellent Prognosis frequent meals with low carbohydrates and high
• Endoscopic mucosal resection (EMR) @ Endoscopic protein/fat
submucosal dissection (ESD)
• Therapeutic  curative for early gastric cancer (gastric CA Late dumping syndrome: occur 1-2hr after a meal
superficial to muscularis propria regardless of LN status) due to reactive hyperinsulinaemia with
hypoglycaemia in response to rapid delivery of
Nutritional support food into small intestine  treat by eating
• Assess: more carbohydrates
 history (significant weight loss)
 physical examination (BMI < 18.5kg/m2), * Pathophysiology of dumping:
 labs (serum albumin <35g/L) With meal  rapid gastric emptying leading to:
• Pre-operative nutritional support significantly improves 1. Hyperosmolar jejunal chime  intraluminal
morbidity and mortality in malnourished surgical patient fluid sequestration leading to:
 at least 7 days to have clinical benefit a. Decreased blood volume 
• Route: enteral or parenteral hypotension, tachycardia,
• Estimated nutritional requirements = 20-35kcal/kg/day palpitations, dizziness
b. Abdominal bloating  diarrhoea
Chemotherapy / Radiotherapy 3. Inappropriate gut hormone release 
1. Neo-adjuvant Therapy – significant improvements in 5 vasomotor and GI symptoms
year survival rates  chemotherapy regimen: 5FU, 4. Rapid glucose absorption  inappropriate
epirubicin, cisplatin (potential for increasing resectability insulin release  late hypoglycaemia
rates  down-staging)
2. Adjuvant Therapy – improvement in overall disease free Nutritional deficiency
survival rates with resected gastric cancer treated post- - Iron deficiency – mixed picture
operatively with 5-FU chemotherapy coupled with 1. Loss of intrinsic factor  B12 deficiency
radiation therapy 2. Decreased conversion of iron from Fe3+
to Fe2+ by gastric acid  decreased iron
Surgery choice: absorption in terminal ileum
Subtotal leaves a small portion of proximal stomach - Need to supplement with B12 injections and Fe
gastrectomy – easier to anastomose to jejunum than supplements
oesophagus since oesophagus does not
have serosa (higher risk of leak) a/w less
morbidity, better functional outcome
(some residual reservoir fx preserved) 
preferred option for distal tumours
Total resection of choice for proximal tumours
Gastrectomy (fundus, cardia, body) as well as diffuse-
type tumours and cardio-oesophageal
junction tumours
.

UiTM 26
TEMPLATE CASE BASED SCENARIO EXAMINATION (CBSE) 20/21

UiTM 27
TEMPLATE CASE BASED SCENARIO EXAMINATION (CBSE) 20/21

LGIB PAINLESS: COLORECTAL CARCINOMA

CASE SCENARIO
Name Mr Ahmad
Age 58 years old
Race, Gender Malay, male
Underlying
Chief complaint Per rectal bleeding
Duration 6 months
Associated symptoms (if any) + duration

Colorectal CA • Mixed with stool/ malaena


• Abdominal mass/ anaemic symptoms
• Alternating bowel habits/ tenesmus
• Constitutional symptoms
Diverticulosis (diverticulitis) • Massive PR Fresh blood
• Elderly
• May cause haemodynamic instability
Angiodysplasia • Massive PR bleed
• Elderly
• May cause haemodynamic instability
Haemorrhoid • Fresh blood after defecation
• Painful/ painless  depends on internal/ external haemorrhoid
• Mostly a/w with low intake of fibre  constipation
• RF: prolonged straining/ constipation/ diarrhea/ hereditary
• May have high increase intraperitoneal pressure
Anal fissure • Painful PR bleed
• a/w constipation
Inflammatory bowel disease • Painless per rectal bleeding
• a/w bloody diarrhea
• blood mixed with mucus
• may have other systemic symptoms

UiTM 28
TEMPLATE CASE BASED SCENARIO EXAMINATION (CBSE) 20/21

PART 1

HISTORY
MUST GET GOOD TO GET
EXPLORE CHIEF COMPLAINT COMPLICATIONS FROM THE DISEASE

Per rectal bleeding (LGIB) Complication 4B: block/ bleeding/ burrow/


S - of luminal burst
O 1st timer/ recurrent? Every bowel motion organ Block : IO: constipation/
abdominal distension/ vomiting/
C Painful/ painless abdominal pain
R - Burrow: fistula between adjacent
A a/w bloody diarrhea ? watery diarrhea organ: bladder  feces/ flatus
Right: abdominal pain/ mass  colicky/ (pneumaturia) through urethral
crampy/ anemic sx orifice
Left: alternating bowel habit/ tenesmus/ Burst: peritonitis; sudden/ acute
constipation, diarrhea/ pruritus onset diffuse @ localized
T Nature: abdominal pain
blood alone @ blood with mucus @ blood Metastatic • Bone: bone pain, fracture
after defecation @ blood mixed with stool symtoms (osteoporosis),
@ blood streak on stool hypercalcemia sx
E • Lung: dyspnea, pleuritic
S - chest pain, haemoptysis
Colour Fresh blood/ malaena/ dark red • Liver: jaundice, RUQ pain,
Amount Profuse/ scanty ascites
• Brain: seizure, headache,
ASSOCIATED SYMPTOMS drowsy (increase ICP)
Perianal • lump, pruritus
symptoms • pain/ discharge RISK FACTORS
Constitutional • LOA, Significant LOW
symptoms • Fever MODIFIABLE NON-MODIFIABLE
• Anorexia/ lethargy  Low fibre diet  Family hx of FAP/
Diet history • Low fibre intake  High consumption of IBD/ colorectal
red meat, processed CA
• High consumption red meat
meat  Previous
Family history • Family history of colorectal CA  Smoking, obesity, radiotherapy for
• Family history of colon polyp  sedentary lifestyle urogynaecological
Anaemic sx • Fatigue  drug: aspirin, NSAIDs CA
• Breathlessness B-blocker
• Palpitation
• Dyspnoea on exertion .
• Fainting
• Light headedness
• Headache
Systemic sx • Eyes: uveitis
(particularly • Skin: erythema nodosum
IBD) • HPB: jaundice
• MSK: arthritis
• Dysphagia/ odynophagia
.

UiTM 29
TEMPLATE CASE BASED SCENARIO EXAMINATION (CBSE) 20/21

PHYSICAL EXAMINATION
MUST GET GOOD TO GET
General symptoms: Peritonism sign
 Body built  cachexic
 BMI: low  Abdominal tenderness/ guarding
 Visible peristalsis
Vital sign: BP, PR, RR, temperature  esp BP, PR, temp:  Absent bowel sound
hypotension/ tachycardia/ slight rise

Hydrational  Loss of weight. Metastatic sign  bone tenderness


status  Sunken eyes.  pleural effusion
 Dry tongue and inelastic skin  hepatomegaly
 Dry mucosa Assessment of  BMI
Anaemic  Palmar pallor nutrition  Waist-hip ratio
sign  Conjunctival pallor (anthropometry)  Mid-arm circumference
Muscle  Muscle wasting of masseter, Dynamic hand  Reduce
wasting temporalis grip test
 Edematous (d/t protein loss)
Abdominal  Cough impulse
examination  Tenderness/ guarding
 Mass of abdomen
 Ascites
 Previous scar
Stoma  site: LIF (colostomy), RIF (ileostomy)
examination  type: permanent/ temporary
 lumen: double loop, single loop
 content: semi-solid/ solid fecal
material/ bilious content
 function: well-function/ vapour
Digital rectal  Content: blood with stool @
examination malaena @ mucus @ pus
 Consistency stool: Impacted stool/
soft stool
 Skin tags/ masses/ polyps/ prolapse
swelling
 Fistula tract/ opening
 Anal tone; Palpate for prostate

PART 2

FULL DIAGNOSIS
Presentation Colorectal CA
Secondary to
With underlying (if any)

UiTM 30
TEMPLATE CASE BASED SCENARIO EXAMINATION (CBSE) 20/21

INVESTIGATION
MUST GET GOOD TO GET
DIAGNOSTIC TO ASSESS COMPLICATION

Bedsite Nutritional assessment


Proctoscopy TRO mass at rectum/ hemorrhoid Biochemical test
(usually at 3, 7, 11 o’clock) Full blood count  Low Hb
(FBC)
Imaging Liver function test  Low albumin
Colonoscopy 1. To see the primary lesion (LFT)  ALT, AST (Liver
(diagnostic and 2. To see the extend of primary metastasis)  40%
therapeutic ) lesion mets baru ade
3. To look for secondary lesion deranged LFT; but
4. For diagnostic measures  doesn’t mean normal
do biopsy sent for HPE LFT xde liver mets
5. For therapeutic measure; ie
BUSE  Electrolyte imbalance
polypectomy
Barium enema To look lesion in colon Carcinoembryonic  surveillance of the
Abdominal X- look for IO , cecum distention antigen (CEA) level disease
ray Immunological
Erect CXR air under diaphragm (perforated FBC  WCC: low
tumor) Complement level  Low C3, C4

Staging
CT TAP Staging of the disease  look for
metastatic spread
Endoscopic Loco-regional staging for rectal
rectal U/S tumor
Positron accurate staging (but limited
emission opportunity in Malaysia)
tomography
with CT (PET-
CT)
MRI rectum: in suspected rectal CA
Chest X-ray Look for lung metastasis 
cannon-ball appearance
.

UiTM 31
TEMPLATE CASE BASED SCENARIO EXAMINATION (CBSE) 20/21

MANAGEMENT
MUST GET GOOD TO GET
DEFINITIVE
Early stage : (stage I)
Depends on TNM staging  Polypectomy via colonoscopy
 Endoscopic mucosal resection

Minimally invasive: (stage II)


 Laparoscopic partial colectomy with/without
colostomy

Advanced : (stage III, IV)


 Symptomatic surgery
 Therapeutic not curative ( to relieve signs and
symptoms )
 Chemotherapy
 Radiotherapy
 Targeted drug therapy
 Immunotherapy
 Palliative therapy

Hartman’s procedure  to allow healing post-


surgery when immediate anastomosis not possible
(closure of rectal stump + formation of end
temporary end colostomy)

Rectal CA

Local excision (only for small


tumours)

Low /ultra-low upper 2/3 of the


anterior resection rectum (so that we can
(AR) ± temporary try to preserve anus)
colostomy
(Hartmann’s) from anal verge:
High Above 10cm
Low Between 6-
10cm
Ultra- Less than 6cm
low
 distal 1/3 of the
Abdominoperineal rectum
Resection (APR)  within 5cm from
with permanent anal verge
colostomy

UiTM 32
TEMPLATE CASE BASED SCENARIO EXAMINATION (CBSE) 20/21

Colon CA management Rectal CA management

T1, T2 (Duke A) surgery T1, T2 Duke A surgery


(stage I) (stage I)
T3, T4 (Duke B) surgery ± adjuvant T3, T4 Duke B Neoadjuvant
(stage II) chemotherapy (if has high (stage II) chemoradiotherapy
risk features) If any N Duke C Followed by:
If any N (Duke C) surgery + adjuvant (stage III)  Surgery
(stage III) chemotherapy  Adjuvant
If any M (Duke D) palliative surgery/ chemotherapy
(stage IV) chemotx If any M Duke D palliative surgery/
. (stage IV) chemotx
.

Carcinoma CECUM Right hemicolectomy


Carcinoma of hepatic flexure Extended right hemicolectomy
Carcinoma of Pelvic colon Left hemicolectomy

UiTM 33
TEMPLATE CASE BASED SCENARIO EXAMINATION (CBSE) 20/21

LGIB PAINFUL:

CASE SCENARIO
Mr Ahmad, 50 years old presented to HS with initial complaint of painful per rectal bleeding for 3 months.

PART 1

HISTORY
MUST GET GOOD TO GET
EXPLORE THE CHIEF COMPLAINT RISK FACTOR
 Painful/ painless 1. Human papilommavirus ( HPV) infection
 Bloody diarrhea? Watery diarrhea? 2. Age > 50 years old
 Colour  fresh blood/ malaena 3. Frequent anal irritation
 Nature  blood alone @ blood with mucus @ blood 4. Anal fistula
after defecation @ blood mixed in stool @ blood 5. Cigarrete smoking
streak on stool 6. Lowered immunity : HIV , organ transplantation ,
 Alternating bowel habit/ tenesmus/ constipation immunisuppressive drugs
 Abdominal pain/ mass colicky/ crampy/
 Nausea/ vomiting/ anemic symptoms
 Diet history: fibre intake, meat intake

Constitutional sx:
 LOA, LOW, fever, fatigue, night sweat

ASSOCIATED SYMPTOMS
 Perianal pain
 Pruritus
 Recurrent discharge/soiling
 Recurrent perianal abscess
 Passage of flatus thru track

DIFFERENTIAL DIAGNOSIS
Anal Carcinoma  Blood steak on stool + tenesmus
Anal abscess
Anal fissure  Tearing pain with defecation
 Painful fresh PR Bleed (usually noted on toilet paper or on surface of stool)
 Severe anal spasm that last for hours afterward
 Perianal pruritus and/or skin irritation
Anal fistula  Perianal pain
 Pruritus
 Recurrent discharge/soiling
 Recurrent perianal abscess
 Passage of flatus thru track

UiTM 34
TEMPLATE CASE BASED SCENARIO EXAMINATION (CBSE) 20/21

PHYSICAL EXAMINATION
MUST GET GOOD TO GET
 Digital rectal examination + Proctoscopy  Anaemic sign: palmar/ conjunctival pallor
Patients often unable to tolerate DRE / Anal  Assessment of nutrition
Speculum  Cachexic
Option: EUA (examination under anaesthesia):  BMI
mostly spinal  Dehydrational status
INSPECTION
Fissure Acute: superficial tear (usually Metastatic sign: bone tenderness, pleural effusion,
posteriorly) – if lateral (consider hepatomegaly
secondary causes)

Chronic:
 Hypertrophied with skin tags
and/or papillae
 Boat shaped
 Punched out
 Exposing internal sphincter
 Sentinel skin tag
 Hypertrophic anal papilla
Abscess  Small
 Erythematous
 Well-defined
 Fluctuant
 Subcutaneous mass near the anal
orifice
 Tender swelling
 Purulent discharge
Fistula  Fistula tract/ opening

Carcinoma Lump or swelling near the anus

 Content: blood with stool @ malaena @


mucus @ pus
 Consistency stool: Impacted stool/ soft stool
 Skin tags/ masses/ polyps
 Anal tone; Palpate for prostate

PART 2

FULL DIAGNOSIS
Presentation
Secondary to
With underlying (if any)

UiTM 35
TEMPLATE CASE BASED SCENARIO EXAMINATION (CBSE) 20/21

INVESTIGATION
MUST GET GOOD TO GET
DIAGNOSTIC TO ASSESS COMPLICATION
Full blood count - Hb
Fistula Renal profile - BUN
 Endoanal U/S (H2O2 aided for hyperechoic Coagulation profile - coagulopathy
effect) – to view course of fistula tract Liver function test
 MRI – able to visualise entire pelvis, beyond the Stool culture, ova and parasite testing
sphincter complex (gold standard) Group cross match
 CT/fistulography (in emergency situation) – for
complex fistulas / unusual anatomy

Suspected anorectal cancer


 Biopsy of the mass : Histopathology evaluation –
to differentiate SCC or Adenocarcinoma
 CT Scan, MRI : Staging and assess metastatic
spread
 PET scan

MANAGEMENT
MUST GET GOOD TO GET
ANORECTAL ABSCESS

ANAL CARCINOMA
 Squamous Cell Carcinoma
For non metastatic anal cancer : Concurrent chemotherapy ( 5 –fluoroucil + mitomycin ) and radiotherapy .
If failed , disease become progressive : Surgical method
- Wide local incision with 1 cm margin of normal tissue
- Abdominoperineal resection with colostomy (if tumor > 50% of anal canal)
For advanced stage ( Stage IV ) : Systemic chemotherapy ( 5 - fluoroucil + cisplatin)

 Adenocarcinoma
Surgical : Abdominoperineal resection ( APR )
Chemoradiotherapy as adjunct pre and post operative

UiTM 36
TEMPLATE CASE BASED SCENARIO EXAMINATION (CBSE) 20/21

For patients who have a complete remission from initial chemoradiotherapy, suggest the following every 3 to
6 months for 5 years:
 Digital rectal examination
 Anoscopy
 Inguinal node palpation
 CT TAP annually for 3 years (stage T3-4)

FISTULA
1. Fistulotomy (for simple, short tracts) – cut & lay open tract to heal
2. Fistulectomy – core along tract & remove tract entirely
3. Seton – for complex, long, high tracts

FISSURE

UiTM 37
TEMPLATE CASE BASED SCENARIO EXAMINATION (CBSE) 20/21

PERIPHERAL VENOUS DISEASE (PVD)

CASE SCENARIO
Name
Age 45
Race, Gender Women
Underlying
Chief complaint Non healing Ulcer right medial malleolus
Duration 3 months
Associated symptoms (if any) + duration Symptoms of leg fullness
Aching discomfort
Heaviness
Nocturnal leg cramps
Bursting pain upon standing.

PART 1

HISTORY
MUST GET GOOD TO GET
EXPLORE THE CHIEF COMPLAINT RISK FACTORS
MODIFIABLE (SECONDARY)
ASYMPTOMATIC 1. Occupation – requiring long periods of
Cosmetic concern standing
2. Increased abdominal pressure –
SYMPTOMATIC constipation, chronic cough, etc.
1. Nonspecific pain, tingling, aching, burning, 3. Pelvic tumour or other lesion compressing
muscle cramps on the deep veins
2. Leg heaviness
3. Swelling 4. Parity (Pregnancy)
4. Itching skin (pruritus) 5. Weight (increased BMI)
5. Restless leg / Leg tiredness 6. Posture – crossing legs all the time
6. Skin changes 7. Smoking
7. Paresthesia 8. Presence of AV shunt
8. Night cramps
9. Ulceration
NON – MODIFIABLE (PRIMARY)
AGGRAVATING FACTORS 1. Advancing age
with heat, worsen throughout course of day 2. Family history:
(esp. if stand for long periods) 1 parent (50% risk)
Both parents (up to 80% risk)
RELIEVING FACTORS 3. Prior venous thrombosis
resting or elevating legs or wearing elastic stockings
COMPLICATIONS FROM THE DISEASE
*refers to the stage
ASSOCIATED SYMPTOMS 1. Thrombophlebitis
2. Bleeding
3. Hyperpigmentation
4. Eczema
5. Ulceration

UiTM 38
TEMPLATE CASE BASED SCENARIO EXAMINATION (CBSE) 20/21

CAUSES

COMPLICATION FROM THE TREATMENT


PRIMARY
IDIOPATHIC / MULTIFACTORIAL
may be related to posture and components and DIFFERENTIAL DIAGNOSIS
structure of the vein wall
Ulcer secondary to PVD
CONGENITAL
 Defects in the sapheno – femoral valve
(cause GSV varicosities) Ulcer secondary to Diabetic neuropathy
 Defects in sapheno – popliteal valve (cause
SSV varicosities)
 Defects in the perforators valve Decubitus (pressure) ulcer
secondary to trauma
SECONDARY
1. Proximal vein obstruction
 Mechanical factors : pregnancy, pelvic Ulcer
mass secondary to vasculitis
 DVT

2. Destruction of valves by thrombosis Marjolin ulcer in Basal cell carcinoma


3. Increase in flow and pressure secondary to AVF
4. Superficial Thrombophlebitis
5. Hormonal : high estrogen state
6. Collagen vascular disease (Marfan syndrome)
7. Hypoplasia or aplasia of the venous system
(Kippel – Trenaury syndrome)

UiTM 39
TEMPLATE CASE BASED SCENARIO EXAMINATION (CBSE) 20/21

PHYSICAL EXAMINATION
MUST GET

SPECIAL TEST
Need to know 1. Tourniquet test **
2. Tredelenburg test **
3. Perthes’ walking test **

Others 1. Pratt’s test


2. Fegan’s test
3. Short saphenous vein incompetence
4. Morrissey’s cough impulse test

UiTM 40
TEMPLATE CASE BASED SCENARIO EXAMINATION (CBSE) 20/21

GOOD TO GET

UiTM 41
TEMPLATE CASE BASED SCENARIO EXAMINATION (CBSE) 20/21

COMPLETING THE EXAMINATION


Use of a handheld Doppler probe to detect incompetence
 Doppler probe is placed in the popliteal fossa over small saphenous vein
 Squeeze the calf to empty the veins – should hear a whoosh as blood flows through the small saphenous
vein
 When the calf is relaxed there should not be any sound – a second whoosh indicates reflux of blood i.e.
there is valvular incompetence

UiTM 42
TEMPLATE CASE BASED SCENARIO EXAMINATION (CBSE) 20/21

PART 2

CASE SCENARIO
Name
Age 45
Race, Gender Women
Underlying
Chief complaint Non healing Ulcer right medial malleolus
Duration 3 months
Associated symptoms (if any) + duration Symptoms of leg fullness
Aching discomfort
Heaviness
Nocturnal leg cramps
Bursting pain upon standing.

FULL DIAGNOSIS
Presentation
Secondary to
With underlying (if any)

INVESTIGATION
MUST GET GOOD TO GET
DIAGNOSTIC
Venous Duplex ultrasound
Indication 1. Recurrent varicose veins
2. History of superficial thromobophlebitis or DVT
3. Complications of CVI: Venous eczema, Hemosiderin staining, Venous ulceration,
Lipodermatosclerosis

Method 1. Ask for SFJ and SPJ reflux, perforator, deep venous incompetency & DVT screen
2. Can delineate deep and superficial venous systems and locate sites of incompetence
3. Valve closure time should be assessed, usually within the GSV with times >0.5 sec =
abnormal
4. Exclude presence of deep vein thrombosis – stripping is contraindicated

Others: continuous wave Doppler

COMPLICATION
FBC Leucocytosis = infection
Serum electrolytes
Serum blood sugar
ABSI Exclude PAD
Biopsy of the ulcer TRO Marjolin ulcer
MRI TRO OM

UiTM 43
TEMPLATE CASE BASED SCENARIO EXAMINATION (CBSE) 20/21

MANAGEMENT
MUST GET GOOD TO GET
DEFINITIVE (SURGICAL)
Indications
1. Bleeding
2. Thromboplebitis
3. Cosmesis – large unsightly varicosities
4. Complications – signs of chronic venous
insufficiency, venous ulceration
CONSERVATIVE 5. Failed medical treatment
Suitable for 6. Symptoms – pain, discomfort
1. Uncomplicated case with mild varicosity
Ablation therapy
2. Asymptomatic cases
3. Absence of SFV incompetence Chemical Sclerotherapy

Thermal 1. Radiofrequency energy


2. Laser energy

Mechanical 1. Vein ligation / stripping


2. Stab phlebectomy (stab
avulsion)
3. Open or endoscopic
perforator ligation

UiTM 44
TEMPLATE CASE BASED SCENARIO EXAMINATION (CBSE) 20/21

PERIPHERAL ARTERIAL DISEASE (PAD)

CASE SCENARIO
Name Mr Jones
Age 60
Underlying Diabetes Mellitus for 20 years/chronic smoker
Chief complaint Presents with right foot pain with dark discoloration of the toes on the same side for 1
month. The affected toes are found to have loss of sensation to touch. Further
questioning reveals that he has been experiencing limitation of walking because of right
leg pain for nearly 6 months. Patient denies hx of trauma to the right leg.
Duration
Associated symptoms (if
any) + duration

PART 1

HISTORY
MUST GET GOOD TO GET
EXPLORE THE CHIEF COMPLAINT ASSOCIATED SYMPTOMS
S Right foot  Numbness, pin and needles pain
O Acute - Embolic cause <2 week  Ulcer
 Progresses over minutes - Often able to  Black discoloration  cyanosis
accurately recall actual moment of onset  Rest pain
 Symptoms more severe and more 1. Pain
dramatic 2. Paraesthesia
 No history of claudication 3. Paralysis
 Known emboli sources: AMI, AF, 4. Perishing cold
Aneurysm 5. Pallor
 No prior vascular procedure 6. Pulselessness

Chronic onset- Thrombotic cause >2 week  Impotence/ failure to achieve erection
1. Last time walk without pain  Leriche syndrome (occlusion of both internal
2. Started experiencing pain iliac arteries)  reduce blood supply to penis
3. Walking distance before develop pain
4. Distance remain the same, increase, Site - claudication
reduced? Quantify More proximal, the higher the level
*Rapid decreasing claudication Aortic occlusion Both limb
(thrombosis/aneurysm/trauma) Iliac artery occlusion Buttock, thigh, calf
C Cramping Superficial femoral Calf
R Distal - proximal pain : sciatica artery occlusion
Proximal - distal : neuropathic
A Intermittent claudication (muscle) Other related history
1. Pain calf muscle  Chest pain
2. Develop only when the muscle is  SOB
exercised  Hx of heart attack
3. Pain disappear when at rest  Loss of consciousness
*differentiate with OA (bone pain) and  Limb weakness
sciatica (pain nerve)
T Continuous rest pain

UiTM 45
TEMPLATE CASE BASED SCENARIO EXAMINATION (CBSE) 20/21

 Awareness of the pain usually start at DIFERRENTIAL DIAGNOSIS


night while sleep 1. Peripheral Arterial Disease
E  Exacerbate by dangling the legs down to Claudication
floor Rest pain
 Does not relieve with PCM Skin changes
S Severe pain  at least 7
2. Neurogenic pain
RISK FACTORS/PMH Bench to bench
1. Smoking
2. Family hx of arterial disease 3. DVT
3. Hypertension Prolonged immobilisation
4. Diabetes Mellitus Long travelling
5. Hyperlipidaemia Malignancy
Past surgical hx
FAMILY HISTORY
 Premature cardiovascular death 4. Chronic venous insufficiency
Non healing ulcer
SOCIAL HISTORY Skin pigmentation
 Occupation (work involve a lot of walking - Lower limb unilateral swelling
postman)
 Breadwinner 5. DFU
 Ground floor/Apartment - Available of lift
 Hobby
 Does he own a car
6. Burger’s Disease/ thromboangitis obliterans
DRUG HISTORY
 Anti hypertensive drug - B blocker
 Anti diabetic drug 7. Raynaud’s Disease
 Lipid lowering drug  Discolouration

COMPLICATION
1. Compartment syndrome
2. Release of substances from the damaged muscle
cells, such as:
a) K+ ions causing hyperkalaemia
b) H+ ions causing acidosis
c) Myoglobin, resulting in significant AKI

UiTM 46
TEMPLATE CASE BASED SCENARIO EXAMINATION (CBSE) 20/21

PHYSICAL EXAMINATION
MUST GET GOOD TO GET
 GENERAL  NEUROLOGY EXAMINATION
1. BMI (Obesity) - claudication worsen as it  Weakness
increase demand of the myocardium  Sensory
2. Vital sign : PR/BP
3. Anemia/polycythemia  CARDIOVASCULAR EXAMINATION
4. Corneal arcus  Risk of arterial embolism with arterial
5. Xanthomata fibrillation (acute)
6. Nicotine stained  Sign of heart failure

 LOWER LIMB EXAMINATION  ABDOMINAL EXAMINATION


Inspection 1. Colour Inspection Scars of previous operation;
2. Trophic changes pulsatile mass
3. Muscle wasting Palpation Masses with expansile pulsation
4. Ulcer/gangrene (aneurysm)  TRO AAA
5. Edema Hepatomegaly (Heart failure)
Palpation 6. Temperature Auscultation Abdominal aortic bruit
7. Tenderness
8. CRT
9. Palpate all pulses -
femoral/PTA/DPA

Auscultation 10. Bruit over femoral artery

Burger’s Test
Normal Remain pink with elevation
Significant Pallor with elevation, in dependent
PAD position will turn pallor
Chronic Intensifies skin color changes due
Arterial to maximally dilated arterioles
Occlusion

PART 2

FULL DIAGNOSIS
Presentation Gangrenous right chronic ischemic leg due to atherosclerosis (PAD)
Secondary to
With underlying (if any) Poorly controlled hypertension and hyperlipidaemia

UiTM 47
TEMPLATE CASE BASED SCENARIO EXAMINATION (CBSE) 20/21

INVESTIGATION
MUST GET GOOD TO GET
DIAGNOSTIC SUPPORTIVE
1. Ankle Brachial Pressure Index (ABPI)
>1.3 Calcification Bedsite
0.9 - 1.1 Normal  ECG - to look for atrial fibrillation
0.7 - 0.9 Mild limb ischaemia
0.5 - 0.7 Moderate Blood
0.3 - 0.5 Critical  Full blood count
<0.3 Irreversible/ulcer/gangrene  Coagulation profile
 Renal profile - AKI
2. Duplex/Doppler US - to confirm the absence of  Lipid profile
pulses.  Blood glucose level
3. CT Angiography  Serum lactate - to assess the severity of
4. Magnetic Resonance Angiography (MRA) - to ischaemia  anaerobic respiration
guide revascularisation if the limb is viable,
where delaying treatment is not threatening to Imaging
limb viability.  Chest xray
5. Digital subtraction angiography
Others
 Treadmill/Exercise Test

ACUTE - RUTHERFORD CLASSIFICATION

UiTM 48
TEMPLATE CASE BASED SCENARIO EXAMINATION (CBSE) 20/21

MANAGEMENT
MUST GET GOOD TO GET
DEFINITIVE - Revascularisation Non-critical vs critical

INITIAL MANEGEMENT - ACUTE LIMB ISCHAEMIA MEDICAL MANAGEMENT


1. Arrange emergency assessment by a vascular specialist 1. Infrequently, anticoagulation alone
2. Systemic anticoagulation with heparin - Bolus of 5000 - 10 may be sufficient if symptoms are
000U of intravenous Heparin mild or cease.
a) Anticoagulants effect : prevents proximal and/or distal
thrombus propagation and preserves the LONG TERM MANAGEMENT
microcirculation. 1. Reduction of the cardiovascular
3. Analgesia: paracetamol and an opioid mortality risk - Lifestyle modification
4. High flow oxygen a) Promote regular exercise,
5. Fluid replacement if dehydrated exercise training  different
jarak
b) Smoking cessation
c) Weight loss
d) Control blood pressure, blood
sugar
e) Modify diet
2. Started on an anti-platelet agent /
anticoagulation
e) Anti-platelet
(clopidogrel/aspirin) 75mg/day
f) Statin: stabilize atherosclerotic
plaques
3. Treat underlying condition e.g. AF
4. If amputation was done
 Occupational therapy
 Physiotherapy
 Long term rehabilitation plan

CTA = computed tomography angiography;


DSA = digital subtraction ultrasound;
DUS = duplex ultrasound.

Surgical - Mandatory in Rutherford 2a - urgent endovascular


revascularisation
If cause embolic If cause thrombotic
 Embolectomy via a  Local intra-arterial
Fogarty catheter thrombolysis
 Local intra-arterial  Angioplasty
thrombolysis - urokinase,  Bypass surgery
alteplase  End-arterectomy
 Bypass surgery (if there is
insufficient flow back)

Non viable - amputation

UiTM 49
TEMPLATE CASE BASED SCENARIO EXAMINATION (CBSE) 20/21

CHRONIC LIMB ISCHAEMIA


-Conservatively :-

Pharmacology
a) Dual Anti-platelet (clopidogrel/aspirin) 75mg/day
b) Statin: stabilize atherosclerotic plaques
c) PDE2 phosphodiasterase inhibitor (cilostazol) : arterial
vasodilator, inhibit platelet vasodilator
d) Analgesic PRN

Surgical
Indication:
 Critical limb ischaemia  when have gangrene
 Failure medical treatment
 Disturb quality of life
1. Transluminal angioplasty & stenting
2. Bypass

UiTM 50
TEMPLATE CASE BASED SCENARIO EXAMINATION (CBSE) 20/21

HERNIA: INGUINAL HERNIA

CASE SCENARIO
Name Mr Aman
Age 44 years old
Race, Gender Malay, male
labor worker, chronic smoker
Underlying
Chief complaint Swelling at left groin
Duration For 1 year
Associated symptoms (if any) + duration

UiTM 51
TEMPLATE CASE BASED SCENARIO EXAMINATION (CBSE) 20/21

PART 1

HISTORY
MUST GET GOOD TO GET
EXPLORE CHIEF COMPLAINT COMPLICATIONS FROM THE DISEASE

Swelling  age of onset Obstructed • Vomiting


S Unilateral/ bilateral • Abdominal distension
Groin  scrotum • Colicky abdominal pain
O Sudden/ progressively • Constipation
Increase in size • Irreducible lump
C Painful/ painless • Lump tender
Strangulated • Painful
R - • Inflamed skin
A a/w nausea/ vomiting/ fever/ ability to pass • Fever
fetus • Edematous
T Sense of dragging/ heaviness Irreducible/ • Cannot be reduced
E Coughing/ straining/ crying/ walking incarcerated • IO symptom: vomiting,
abdominal pain,
S
distension, constipation
Relieves Spontaneous on lying down/ manual
reduction
RISK FACTORS
 Chronic cough (TB, bronchiolitis, asthma)
ASSOCIATED SYMPTOMS  Straining during micturition (bladder
Urinary sx Storage + voiding symptoms: outflow obstruction)/ Constipation
Storage:  History of surgery (appendicectomy)
• Frequency, urgency, nocturia,  Heavy lifting work/ weightlifting
incontinence  History of trauma
 Mass in abdomen/ ascites
Voiding
• Difficulty to urinate, hesistency,
intermittent dribbling
.

UiTM 52
TEMPLATE CASE BASED SCENARIO EXAMINATION (CBSE) 20/21

PHYSICAL EXAMINATION
MUST GET GOOD TO GET
General symptoms: Peritonism sign
 Body built
 BMI: obesity  Abdominal tenderness/ guarding
 Visible peristalsis
Vital sign: BP, PR, RR, temperature  esp BP, PR, temp:  Absent bowel sound
hypotension/ tachycardia/ slight rise

Abdominal  Cough impulse


examination  Mass of abdomen/ ascites/
previous scar
Hernia  Wasting muscle at temporalis,
examination masseter
 Wasting at hypothenar/ thenar
muscle

Palpation during standing

 Feel for temperature, tenderness, consistency


 Confirm inspection finding: site, size, shape, surface, border.
 Ask pt to cough + Palpate cough impulse on coughing on both sides
 How to do it:
Patient turns head to opposite side and coughs. If swelling becomes tense and expands, there is a
positive cough impulse
 Identify pubic tubercle, inguinal ligament relation to swelling  medial @ lateral @ inferior @ superior.
 Perform percussion + auscultation
I
N Palpation during supine
G
 Check reducibility of swelling: ask pt to reduce swelling by himself
U  Listen gurgling sound during reduction  big hernia (bowel content)
I  Perform Deep Ring Occlusion Test (hernia must be completely reducible):
 How to do it:
N  Locate the deep inguinal ring (2.5cm above point midway between ASIS and pubic tubercle)
 Apply pressure to DIR using thumb/ 2 fingers (index and ring fingers)  ask pt to cough 
A observe +ve/-ve bulging on coughing.
L  If no bulging  reconfirm by ask pt to cough WITHOUT apply pressure to DIR
 Results:
1. If NO bulging on coughing onto DIR: indirect inguinal hernia
2. If HAVE bulging medial to occluded finger: direct inguinal hernia

UiTM 53
TEMPLATE CASE BASED SCENARIO EXAMINATION (CBSE) 20/21

UiTM 54
TEMPLATE CASE BASED SCENARIO EXAMINATION (CBSE) 20/21

PART 2

FULL DIAGNOSIS
Presentation Inguinal hernia
Secondary to
With underlying (if any)

INVESTIGATION
MUST GET GOOD TO GET
DIAGNOSTIC TO ASSESS COMPLICATION

Clinical diagnosis  presents as intermittent bulge in Laboratory


groin related to exertion or long periods of standing Full blood  increase in WBC
count (FBC) (incarcerated)
Imaging
Renal profile  increase in BUN and
Abdominal X- TRO intestinal obstruction
creatinine 
ray (small bowel obstruction)
obstruction
Ultrasound To look vascularity
Arterial blood  TRO exclude metabolic
.
gases (ABG) alkalosis and respiratory
acidosis
Imaging
Erect chest X-  to look for
ray pneumoperitoneum
(perforation abdominal
viscus  peritonitis)

UiTM 55
TEMPLATE CASE BASED SCENARIO EXAMINATION (CBSE) 20/21

MANAGEMENT
MUST GET GOOD TO GET
DEFINITIVE CONSERVATIVE MANAGEMENT
• Surgery is the treatment of choice.
• Principle: reduce bowel + removal hernia sac + reinforce 1. Raised intra-abdominal pressure
posterior wall • Weight loss, change jobs, avoid heavy
• Conservative measure only indicated in children lifting
• Treat medical conditions causing
Herniotomy: ligation and removal of the sac only chronic cough, chronic constipation
(children) 2. Truss: for compression of reducible hernia
at deep ring (poor pickup rate)
Herniorrhaphy: herniotomy + repair of the posterior 3. If obstructed/strangulated: NBM, IV drip,
wall of Inguinal canal
NG tube on suction, IV ABx
Hernioplasty: herniotomy + reconstruction of the
posterior wall of IC with synthetic
mesh Delineate emergency mx in acute presentation

SURGERY
SUPPORTIVE
 Immediate exploratory laparotomy in
In acute presentation of strangulation:
emergencies
1. Resuscitation (as most pt presented with
 Peritonitis, clinical instability,
hypotension)  ABC- Airway, Breathing, Circulation
 unexplained leukocytosis
2. Plan for surgery as gangrene can occur as early 5-6
hours first onset of symptom.  acidosis are concerning for abdominal
3. Fluids and electrolytes replacement sepsis, intestinal ischemia, or
4. Nasogastric decompression perforation
5. Manual removal of faeces
6. Broad spectrum antibiotics  Operative decompression may be required
if there is severe dilatation of bowel loop.
 The viability of the involved bowel should
be carefully assessed.

UiTM 56
TEMPLATE CASE BASED SCENARIO EXAMINATION (CBSE) 20/21

BREAST: BREAST CARCINOMA

CASE SCENARIO
A 50 years old woman presented with left breast lump and left axillary lump for 3 months

PART 1

HISTORY
MUST GET GOOD TO GET
EXPLORE THE CHIEF COMPLAIN RISK FACTOR

Breast lump UNOPPOSED ESTROGEN


1. Onset  Early menarche (<12yr)
2. Duration  Late menopause (>55yr)
3. Painless or painful  Late age at first full term pregnancy (first birth after
4. Site 30)
5. Single or multiple  How many children? (nulliparity)
6. Nature of the lump/progression of size,  Hormone Replacement Therapy – for menopause
7. How did the pt notice the lump (incidental or BSE)  Oestrogen based OCP (must be estrogen based)
8. Does the lump come periodically (related to  Age (> 25 years old, peak at 60 years old)
menses), mobile or not  Family history (at least 2 generation) of breast
9. Any other lumps elsewhere- axilla, other breast, cancer or gynaecological cancer
neck  Any associated cancers – ovary, colon, prostate,
gastric, pancreatic
ASSOCIATED SYMPTOM
 Skin changes- swelling, redness PROTECTIVE FACTORS
 Any nipple discharge- colour, amount, is the - Whether patient breastfed her children
discharge truly from nipple (paget disease will
have staining on bra w/o obvious nipple
discharge), unilateral or multi-ductal discharge,
persistent or intermittent
 Nipple changes- nipple retraction, nipple
dimpling
 Any history of trauma
 Any previous breast disease before, previous
mammogram
 Metastatic symptoms- bone pain (spine), jaundice
(liver), SOB and chest pain (pleural effusion
 Constitutional sx- LOA, LOW

UiTM 57
TEMPLATE CASE BASED SCENARIO EXAMINATION (CBSE) 20/21

PHYSICAL EXAMINATION
MUST GET GOOD TO GET
 Cachexia, jaundice, pallor, tachypnea

Inspection
 Any breast asymmetry
 Scars- from prev op or biopsy
 Overlying skin changes- fixation of lump to
the skin, peau de orange, ulcerating,
fungating lump, skin retraction, erythema
 Nipple changes- discharge, deviation,
retraction, puckering of nipple, discoloration,
displacement, dermatitis

Palpation
 Lump- shape, site, size, warmth, tender,
surface, consistency, margin, fluctuance,
tethered to the skin, mobile, fixation to
pectoralis major muscle
 Axillary LN- site, size, mobility, tenderness,
consistency
 Palpate 5 groups of axillary LN- anterior,
posterior, medial, lateral, apical

Complete examination
 Examine supraclavicular and cervical LN
 Lungs- pleural effusion
 Percuss spine for bony tenderness
 Examine abdomen- hepatomegaly

DIFFERENTIAL DIAGNOSIS
Breast abscess Painful lump
Redness, warmth, swelling, and tenderness in an area of the breast;
Fever
Phyllodes tumour Small mass may rapidly increase in size in the few weeks
Ulcer
Fibroadenoma Painless lump
Fat necrosis Painless, irregular, firm lump associated with thickening and retraction of the skin

UiTM 58
TEMPLATE CASE BASED SCENARIO EXAMINATION (CBSE) 20/21

PART 2

CASE SUMMARY
A 50 years old woman presented with left breast lump and left axillary lump for 3 months. She is nulliparous and
has family history of breast cancer. On examination, the lump was hard, non-tender, immobile, 4 cm lump
which situates at upper outer quadrant of left breast with mobile palpable nodes at the left axilla.

FULL DIAGNOSIS
Presentation Breast lump
Secondary to Invasive breast carcinoma
With underlying (if any)

INVESTIGATION
MUST GET GOOD TO GET
DIAGNOSTIC TO ASSESS COMPLICATION

Triple assessment
1. Clinical examination
2. Radiological assessment
 Mammogram or ultrasound
 Mammogram- age >40- more fat,
less dense breast tissue
 Malignant features-
microcalcifications, spiculated mass,
architectural distortion, irregular
margin, nipple changes
3. HPE assessment
 FNAC- less invasive, less painful,
smaller wound, does not LA but
cannot differentiate between in situ
cancer and invasive cancer
 Core biopsy- more invasive, need LA,
larger wound, can differentiate
between in situ cancer and invasive
cancer
TRO metastatic spread
 CXR- pleural effusion
 LFT
 Bone scan
 PET scan
 CT or MRI brain

UiTM 59
TEMPLATE CASE BASED SCENARIO EXAMINATION (CBSE) 20/21

MANAGEMENT
MUST GET GOOD TO GET
DEFINITIVE CONSERVATIVE (Lifestyle, diet etc)

Pt education and counselling (breast conserving or


mastectomy, risk of recurrent).

Surgery
1. Wide local excision (tumour <4 cm) or simple
mastectomy
2. Axillary LN- sentinel node biopsy (SLNB) or
axillary clearance
3. Breast reconstruction-
prosthesis/implant/muscle flap (latissimus dorsi
or rectus abdominis ms)

Radiotherapy
1. Local chest wall radiation therapy
2. To axilla

Hormonal therapy
1. For ER/PR +ve- tamoxifen (SERM) or aromatose
inhibitors

Chemotherapy
1. Neoadjuvant
2. Adjuvant
3. Palliative

Targeted therapy
Herceptin/trastuzumab- for HER2 R +ve - IV monthly for
12 months

UiTM 60
TEMPLATE CASE BASED SCENARIO EXAMINATION (CBSE) 20/21

UiTM 61
TEMPLATE CASE BASED SCENARIO EXAMINATION (CBSE) 20/21

UiTM 62
TEMPLATE CASE BASED SCENARIO EXAMINATION (CBSE) 20/21

UiTM 63
TEMPLATE CASE BASED SCENARIO EXAMINATION (CBSE) 20/21

THYROID SWELLING: TYHROID CARCINOMA

CASE SCENARIO
Name Mr Aman
Age 44 years old
Race, Gender Malay, male

Underlying
Chief complaint Neck swelling
Duration For 1 year
Associated symptoms (if any) + duration

Thyroid carcinoma

MNG

Toxic MNG

Thyroiditis

Thyroid adenoma

UiTM 64
TEMPLATE CASE BASED SCENARIO EXAMINATION (CBSE) 20/21

PART 1

HISTORY
MUST GET GOOD TO GET
EXPLORE CHIEF COMPLAINT COMPLICATIONS FROM THE DISEASE
Local • Pain  bleeding into cyst result
Swelling  age of onset in sudden increase in size + pain
Site Anterior? Posterior? • Compressive Symptoms  3D
Size Increase in size  diffuse/ one sided difficulty swallowing, difficulty
 sudden/ progressive breathing, hoarseness of voice
 Earlier size? Current size? (benign  never compress the
O Sudden/ progressively recurrent laryngeal nerve)
C Painful/ painless • Fixed Gaze  risk of optic nerve
compression (surgical
R Stage of disease – ulcer? Changes of emergency)
skin? Erythematous? Systemic • High Output Cardiac Failure – ask
A Hypo/ hyperthyroidism symptoms about dyspnoea, effort tolerance
• Thyroid Storm or Thyrotoxicosis
T
Crisis (metabolic + hemodynamic
E instability)
S
Duration
RISK FACTORS
ASSOCIATED SYMPTOMS
Hyperthyroidism  Hyperactive Hx of • type I DM
symptoms  Easily Irritable autoimmune • SLE, RA,
 Insomnia or Anxiety • pernicious anemia (a/w
 Depression (elderly) Graves and Hashimoto’s
Hx of CA • metastatic disease to thyroid
 Weight loss despite increased elsewhere • lymphoma
appetite • papillary cancer a/w familial
 Heat intolerance polyposis syndromes  ask
 Increased sweating about GI polyps/cancers
 Diarrhoea Hx of • long-standing MNG can
 Palpitations, Tremors thyroid progress to lymphoma
 Oligomenorrhoea and loss of disease
libido Occupation • exposure to ionizing
hypothyroidism  Tiredness / Lethargy radiation (papillary cancer
symptoms  Mood change including risk ↑3x)
depression Family hx of • ~20% of medullary cancers
thyroid CA are familial (MEN2, AD
 Weight gain despite decreased inheritance),
appetite Cold intolerance • ~ 5% of papillary cancers
 Constipation (FAP)
 Bradycardia Social • Smoking
 Menorrhagia history • Drug history: Amiodarone
(cardiac arrhythmia) /
Compressive  Dysphagia
Lithium (bipolar dz)
symptoms  Dyspnea
 Dysphonia .

UiTM 65
TEMPLATE CASE BASED SCENARIO EXAMINATION (CBSE) 20/21

Metastasis Sx • Bone: bone pain, fracture


(osteoporosis), hypercalcemia sx
• Lung: dyspnea, pleuritic chest
pain, haemoptysis
• Liver: jaundice, RUQ pain,
ascites
• Brain: seizure, headache,
drowsy (increase ICP)
.

PHYSICAL EXAMINATION
MUST GET GOOD TO GET
General symptoms: THYROID STATUS
 Cachexic: anxious
 Lethargy  looking thyroid FACE Expression  staring, unblinking
(hyperthyroid); lethargic, apathetic
(hypothyroid)
Vital sign: BP, PR, RR, temperature  esp BP, PR, temp:
hypotension/ tachycardia/ slight rise Complexion  dry, ‘peaches-and-cream’
complexion, loss of outer third of eyebrows
AIM: thyroid ± lymphadenopathy ± features of (hypothyroid)
malignancy ± compressive symptoms ± current thyroid
status (hyper / hypo / euthyroid) Eyes*
a) Lid lag (eyelid lags behind eye when
THYROID patient follows your finger downward)
INSPECTION  Any swelling/ scars? (any b) Lid retraction (sclera visible between
transverse incision in a skin crease, upper limbus of iris and upper eyelid)
2FB above suprasternal notch) c) Exophthalmos (sclera visible between
 Any skin changes over the mass? lower limbus + lower eyelid)
 Any stigmata of hyperthyroidism d) Chemosis (oedema and erythema of
(i.e. agitation) or hypothyroidism conjunctiva) – if present consider using
(i.e. bradykinesia) fluorescein to
 Check for plethora of face, e) look for corneal ulceration
distended neck veins – d/t to f) Ophthalmoplegia (restriction eye
compressive nature of mass (but movements; ask about diplopia!)
rarely seen). g) Proptosis (look from above patient’s
head – eye visible over supraorbital
Differentiate  Check if mass moves on ridge)
ddx swallowing by asking patient to
take a sip of water
UPPER 1. Examine for Proximal myopathy
 Check if mass moves on protruding
LIMB 2. Fine postural tremor – accentuate by
the tongue
placing a sheet of paper on hands
 Check if mass moves on
3. Reflexes (elbow reflex)
swallowing or tongue protrusion
again with hands palpating thyroid

UiTM 66
TEMPLATE CASE BASED SCENARIO EXAMINATION (CBSE) 20/21

Palpate Characteristics of lump and PALM 1. Palms – ↑sweating, palmar erythema


thyroid from surrounding skin: (hyperthyroidism), areas of vitiligo
behind a) site (anterior triangle), (a/w AI disorders, i.e. Graves’ disease)
b) size (discrete nodule/ multinodular 2. Feel pulse – tachycardia, AF (more in
enlargement or diffuse toxic MNG than Graves’), bradycardia
enlargement? (hypothyroidism)
c) consistency (soft, cystic, hard, 3. Nails – thyroid acropachy (clubbing),
multinodular?), onycholysis – plummer’s nail
d) mobility (fixed to skin? Fixed to OTHER 1. Legs for pretibial myxoedema*
underlying structures?), (specific for Graves’ (3%)/ hypothyroid)
e) tenderness. 2. Reflexes (ankle jerk) – slow to relax in
hypothyroidism
Palpate lymph nodes – submental, 3. Offer to check thyroid status and ask
submandibular, pre/post auricular, patient about compressive symptoms.
down SCM (II, III, IV), supraclavicular,
occipital
a) Separate & tender (reactive
hyperplasia)
b) Hard & clustered (carcinoma)
c) Soft and rubbery (lymphoma)

Feel the carotid pulsation on both side


– are they well felt?
Around the 1. Palpate for tracheal deviation
thyroid 2. Percuss manubrium for signs of
any retrosternal extension
3. Auscultate of any neck bruit (using
the bell) – specific for Graves’
.

PART 2

FULL DIAGNOSIS
Presentation Thyroid carcinoma
Secondary to
With underlying (if any)

UiTM 67
TEMPLATE CASE BASED SCENARIO EXAMINATION (CBSE) 20/21

INVESTIGATION
MUST GET GOOD TO GET
DIAGNOSTIC TO ASSESS COMPLICATION

1. Thyroid Function Test: to look for thyroid status Laboratory


2. Ultrasound of Thyroid: to look for malignant Full blood  TRO Anaemia
features count (FBC)
3. Fine Needle Aspiration Cytology (FNAC) of Thyroid: Renal profile  Electrolyte imbalance
classify into Bethesda classification
Coagulation
 Cannot differentiate follicular adenoma/follicular
profile
carcinoma as the mark of malignant disease is
Arterial blood  TRO exclude metabolic
capsular invasion (FNAC provides limited
gases (ABG) alkalosis and respiratory
assessment of architecture)
acidosis
 Need to assess the WHOLE capsule for invasion
Imaging
(hence FNAC, Incisional biopsy/ frozen section
Bone scan  to look for bone metastasis
cannot be used for diagnosis)
4. Anti-thyroid antibodies:
PET scan  more accurate diagnosis
 thyroid peroxidase antibodies (TPO)  grave @
(functional staging)
hashimoto thyroiditis
 thyroglobulin antibodies (Tg)  hashimoto
Chest-X-ray  to look for retrosternal
disease
extension
 thyroid-stimulating hormone (TSH) receptor 
graves’ disease ENT  injure opposite recurrent
examination laryngeal nerve as that can
Clinical diagnosis of vocal cord cause bilateral vocal cord
palsy
Laboratory
Baseline tumor marker: thyroglobulin except
medullary CA (use calcitonin, CEA)
Thyroglobullin surveillance for differentiated
thyroid CA (follicular/ papillary
CA)
Serum Calcitonin Surveillance for medullary thyroid
assay, CEA CA
Imaging
CT scan/ MRi for disease staging + to confirm
sub-sternal/ retrosternal
extension
Radioisotope using I123 @ using (Technecium-
scan 99); not using I-131 since it is
used in Radioactive iodine
therapy

UiTM 68
TEMPLATE CASE BASED SCENARIO EXAMINATION (CBSE) 20/21

MANAGEMENT
MUST GET GOOD TO GET
DEFINITIVE Delineate emergency mx in acute presentation

Differentiated thyroid CA Palliative therapy for compressive effect


1. Surgical resection with TOTAL THYROIDECTOMY with (particularly for anaplastic CA)
LN clearance  Chemotherapy to shrink tumor
 LN clearance: tracheo-oesophageal nodes cleared (doxorubicin, cisplatin)
(central compartment clearance – level VI), and  Tracheostomy
lateral neck dissection (levels II, III, IV) performed  External beam RT: not surgical candidates
for all patients with biopsy proven lymph node
metastasis

2. Adjuvant therapy  RAI -131
 Radioablation iodine (RAI) at 4 weeks after TT while
pt is hypothyroid to ensure iodine uptake

SUPPORTIVE

3. Life-long suppression with L-thyroxine to decrease


recurrence and improve survival

Medullary CA  Surgical resection


 Aggressive resection – total thyroidectomy with level VI
nodes (central
 compartment) clearance
 Selective Neck Dissection indicated for conically involved
ipsilateral cervical lymph nodes
 Sampling of cervical and mediastinal nodes and modified
dissection where positive
Follow-up
 Thyroxine replacement (not for TSH suppression but to
maintain euthyroid state)
 Serum calcitonin and CEA six mths after surgery (if
normal, considered cured – 5% 5yr recurrence)

UiTM 69
TEMPLATE CASE BASED SCENARIO EXAMINATION (CBSE) 20/21

Approach to Abdominal Pain: Acute Pancreatitis

CASE SCENARIO
Name
Age
Race, Gender
Underlying
Chief complaint Epigastric pain
Duration
Associated symptoms (if any) + duration

PART 1

HISTORY
MUST GET GOOD TO GET
EXPLORE THE CHIEF COMPLAINT DIFFERENTIAL DIAGNOSIS

Epigastric pain 1. Gastric causes- PUD


S Epigastric - Epigastric pain
O Acute/Sudden - Hematemesis
C Dull/Boring
R Back 2. Hepatobiliary causes- Hepatitis, GB/CBD
A disease
T Constant/Continuous
May last for several days
E Exaggerated by lying supine 3. Perforated viscus
Relieved by sitting up or leaning forward - Diffuse abdominal pain
S
4. Medical causes- MI, DKA, Lower lobe
ASSOCIATED SYMPTOMS pneumonia
1. N&V

RISK FACTORS (“I GET SMASHED”)


I Idiopathic COMPLICATIONS FROM THE DISEASE
G Gallstone
disease
E Ethanol Chronic alcohol abuse:
Habitual over 5-15 years
COMPLICATION FROM THE TREATMENT
Recent alcohol abuse:
Weekend binging habit
T Trauma Blunt trauma
S Steroids
M Mumps and Mumps: Viral fever +
other infections Bilateral jaw
pain/swelling

VZV
CMV
Mycoplasma

UiTM 70
TEMPLATE CASE BASED SCENARIO EXAMINATION (CBSE) 20/21

Parasitic infections
A Autoimmune SLE
Sjogren’s syndrome
S Scorpion toxin Organophosphate
and other poisoning
toxins
H HyperCa+/
HyperTG
E ERCP
D Drugs “SAND”

Sulfamethoxazole-
trimethoprim

Azathioprine
NSAIDs
Diuretics
Rare Neoplasm Pancreatic/ Ampullary
tumor
Congenital Pancreas divisum
Genetics

UiTM 71
TEMPLATE CASE BASED SCENARIO EXAMINATION (CBSE) 20/21

PHYSICAL EXAMINATION
MUST GET

General
Position
- Lying motionless Diffuse peritonitis
- Sitting up/ Leaning forward Pancreatitis
Small red tender nodules on skin and legs Subcutaneous fat necrosis

Vitals
Tachycardia
Hypotension
Low grade fever
Tachypnea ARDS
- Auscultate lungs: Creps, Reduced air
entry

Abdomen
Abdominal distension
Tenderness
Palpable mass Pseudocysts
Signs of peritonism Rebound tenderness
Guarding
Rigidity
Signs of hemorrhagic pancreatitis Grey-turner sign
- Develops within 48 hours after onset of
symptoms Cullen’s sign

Fox’s sign
Diminished/ Absent bowel sounds Paralytic ileus from diffuse peritonitis

PART 2

FULL DIAGNOSIS
Presentation
Secondary to
With underlying (if any)

UiTM 72
TEMPLATE CASE BASED SCENARIO EXAMINATION (CBSE) 20/21

INVESTIGATION
MUST GET GOOD TO GET
DIAGNOSTIC ASSESS SEVERITY & PROGNOSIS

Serum - Normal: 30-100 U/L Laboratory


amylase - ≥3x upper limit FBC Leukocytosis  Poor prognosis
- Rise within 6-24 hours Hematocrit >44%  Poor
- Normalizes within 3-7 days prognosis
- Elevation >10 days - d/t inadequate fluid
indicates complications resuscitation as
(Pseudocyst formation) evidenced by persistent
- Elevation not a/w dz hemoconcentration at 24
severity or prognosis hours  Increased risk
- Sens 50%, spec 99% for necrotizing
pancreatitis
Serum - Normal: 10-140 U/L RP Urea high
lipase - ≥3x upper limit Electrolyte imbalance
- Rise within 4-8 hours Dehydration
- Stay elevated for 8-14 days Renal impairment
- Useful for delayed acute LFT Elevated ALT, AST
pancreatitis Hypoalbuminaemia
- Sens 64%, spec 97% Blood glucose Hyperglycaemia
LDH Elevated
Urinary - Similar fx to serum lipase CRP Elevated
diastase - Used when serum amylase ABG Negative base excess
is equivocally raised or ECG/ Cardiac MI
normal enzymes

Imaging
UNDERLYING ETIOLOGY Erect CXR Air under diaphragm 
Perforated viscus
LFT ALT >150 mg/dL  Gallstone Pleural effusion
pancreatitis
Complete whiteout  ARDS
Bilirubin >5 mg/dL not fall after 6-12
Supine AXR Localized ileus secondary to
hours  impacted stone in ampulla
inflammation around pancreas:
of Vater
Fasting lipid Hyperlipidaemia i. Sentinel loop sign 
profile Dilated proximal jejunal
Serum Ca/ Hypercalcaemia loop near the pancreas
Mg/ PO4 ii. Colon cut-off sign 
Ultrasound Pancreas is not visualized in up to Distended colon from
abdomen 40% of patients d/t overlying bowel ascending to mid-
gas and body habitus transverse with no air
Visualize biliary tree and gallstones distally
Pancreatic calcification  Chronic
pancreatitis
Contrast- Confirm diagnosis of pancreatitis
enhanced CT if inconclusive blood results
abdomen Complications:
*only after (suspect in persisting organ
aggressive failure, sepsis or clinical
volume

UiTM 73
TEMPLATE CASE BASED SCENARIO EXAMINATION (CBSE) 20/21

resuscitation to deterioration 6-10 days after


diminish risk of admission)
contrast- - Fluid collections
associated
- Necrosis
nephrotoxicity
MRCP Substitute for CT scan if allergic to
contrast or AKI
Visualizing gallstones

UiTM 74
TEMPLATE CASE BASED SCENARIO EXAMINATION (CBSE) 20/21

MANAGEMENT
MUST GET
SUPPORTIVE TREATMENT

1) Resuscitate
Aggressive IV fluid resuscitation Maintain IV volume and allow adequate perfusion to
pancreas and extra-pancreatic organs
2) Monitoring (After resuscitation)
Fluid resuscitation with crystalloids Correct fluid losses in 3rd space
Monitor vitals
- SpO2, HR, BP, Temp
- Urine output (>0.5ml/kg/hr)
- +/- CVP
Monitor electrolytes Every 6-8 hours initially
- Including calcium
Monitor ABG Assess oxygenation and acid base status
Admit
- Mild: General ward
- Severe: HD/ICU
3) NBM (Gastric rest) + Nutritional support
Keep NBM, for at least 2 days or until stable Prevent pancreatic stimulation
Prolonged NBM leads to poorer recovery d/t nutritional
debilitation
Think about NJ feeding or open jejunostomy in severe
pancreatitis
if not, TPN
Insert NG tube Prevent vomiting due to paralytic ileus
If vomiting, do gastric decompression
Proton-pump inhibitors Prevent stress ulcer formation, acid suppression
Start oral feeding early In mild pancreatitis if tolerated
4) Analgesia
Opioid analgesics (Tramadol, Pethidine)
Cannot give
- Morphine Increased tone sphincter of Oddi
- NSAIDs Worsen pancreatitis and cause renal failure (already
decreased renal perfusion in acute pancreatitis)
5) Treatment of fluid and electrolyte abnormalities- Hypocalcaemia, Hypoglycaemia
6) Antibiotics
Prophylactic in severe acute pancreatitis To prevent infection of necrosis
- Carbapenem for 14 days
Not helpful in mild pancreatitis
7) Support organ failure
Manage in surgical HDU
Ventilate with PEEP If hypoxaemia (ARDS)
Dialysis & CVP monitoring Acute renal failure
Fluid resuscitation & inotropes Hypotension

MONITORING & TREATING COMPLICATIONS

UiTM 75
TEMPLATE CASE BASED SCENARIO EXAMINATION (CBSE) 20/21

Local complications
Acute fluid collections (30-50%) 70-80% resolve spontaneously
- d/t increased vascular permeability
Pseudocyst (10-20%) 50% resolve spontaneously if not surgical
- Persistent fluid collection (enzymes, blood, intervention
necrotic tissue) walled off by fibrosis (after 4
weeks)
- Presents as persistent pain, mass on
examination, persistently high amylase or
lipase
- Cx: GOO, infection, peritonitis, hemorrhage
(erosion of splenic vessels)
Sterile pancreatic necrosis (20%) Prophylactic antibiotic (Carbapenem)
- No contract uptake on contrast-enhanced CT Supportive measures, KIV surgery if unstable
scan
Infection (5%)- d/t enteric gram -ve rods
(Enterobacteriaceae)
1) Pancreatic abscess Circumscribed collection of pus w/out pancreatic
tissue
Antibiotics + Drainage (CT guided if possible)
2) Infected pancreatic necrosis Antibiotics
- Gas bubbles on CT scan Wide debridement (Necrosectomy)
- FNA in deteriorating patient with necrosis Lavage and drainage- to decrease infective load
- Gram stain & culture
Chronic pancreatitis (Exocrine & endocrine
insufficiency)

Systemic complications
Peritoneal sepsis
Pancreatic ascites Massive accumulation of pancreatic fluid in peritoneum 
Abdominal compartment syndrome
Pleural effusion
Intra-abdominal haemorrhage Erosion of splenic vessels  Shock
Multiple organ failure ARDS, Acute renal failure, hypovolaemic shock, DIVC
Hypocalcaemia, Hyper/hypoglycaemia

UiTM 76
TEMPLATE CASE BASED SCENARIO EXAMINATION (CBSE) 20/21

Intervention for local complications


ERCP - Not benefit in mild pancreatitis
- Indications:
i. Severe pancreatitis
ii. Ductal stones
iii. Cholangitis
iv. Not response to treatment within 48 hours
- Should be done within first 48-72 hours for maximum benefit
CT guided - Differentiate between sterile and infected necrosis
aspiration of - Consider surgery if patient doing poorly
pancreatic
necrosis
Wide - For infected necrosis
debridement - Followed by lavage and drainage to decrease infective load
(Necrosectomy)
Surgery 1. Infected necrotic pancreas (Mortality 100% without operation)
2. Sterile necrotic pancreas (Necrosectomy)
- Delay surgery till as late as possible for demarcation/organization of necrotic
areas (Repeated surgeries required)
3. Diagnosis uncertainty
4. Complications (Intra-abdominal haemorrhage, pseudocyst)

MANAGEMENT OF ETIOLOGY & PREVENTION OF RECURRENCE

1. Avoid alcohol
2. Stop all offending medications
3. Control hyperlipidaemia

4. ERCP + Endoscopic sphincterotomy


- Done in acute setting (within 72 hours) for severe biliary pancreatitis

5. Cholecystectomy
- For biliary pancreatitis
- Mild: Done in same admission
- Severe: Delay the surgery, as patient may develop complications that require surgical intervention
later. Better to do all surgeries in same operation.

UiTM 77
TEMPLATE CASE BASED SCENARIO EXAMINATION (CBSE) 20/21

HEPATOCELLULAR CARCINOMA

CASE SCENARIO
Name -
Age 60 years old
Race, Gender Indian
Underlying Hepatitis B for 3 years
Chief complaint RUQ abdominal pain
Duration 3 months prior to the admission
Associated symptoms (if any) + duration Vomiting & weight loss

PART 1

HISTORY
MUST GET GOOD TO GET
OVERVIEW : EXPLORE THE CHIEF COMPLAINT
LOCAL SIGNS & SYMPTOMS
1. Upper abdominal pain – dull and
persistent SOCRATES (2o to capsular
distension)
2. prolonged NSAIDs
3. Painful Hepatomegaly
4. Jaundice (5-10%)

Cholestatic invasion/compression of
intrahepatic ducts or extrahepatic
compression by metastatic LN

Have they noticed any change in


their stool recently?
Hepatic a/w pre-existing cirrhosis or acute
flare of chronic hepatitis

5. Early satiety/ vomiting (likely 2o to


compression)
6. Constitutional:
LOW, LOA, malaise (80% of patients)
ASYMPTOMATIC 7. Pyrexia
1. During screening (ultrasound) for chronic (central tumour necrosis)
hepatitis B carrier
2. Incidentally found on imaging of the 8. Hepatorenal syndrome: reduce urine
abdomen output
3. Investigations for liver cirrhosis 9. Coagulopathy
10. When did they last open their bowels or
*HCC need not present with decompensated chronic pass any flatus (wind)
liver disease 11. If they are female, are they pregnant?

UiTM 78
TEMPLATE CASE BASED SCENARIO EXAMINATION (CBSE) 20/21

ASSOCIATED SYMPTOMS Budd-Chiari syndrome


 occlusion of the hepatic, intrahepatic or
FEATURES OF DECOMPENSATED CLD portal vein
1. Jaundice  causing portal hypertension & congestive
2. Coagulopathy hepatopathy
3. Hepatic encephalopathy
4. Hepatorenal syndrome

ABOUT HEPATIC ENCEPHALOPATHY


FEATURES OF PORTAL HYPERTENSION
1. Varices = UGIB (hemetemsis & melena) STAGES : WEST HAVEN CLASSIFICATION
2. Caput medusa
3. Ascites
4. Splenomegaly

CAUSES

FEATURES OF METASTASES
Brain Confusion
Bone Backpain
Lung SOB, hemoptysis
Liver -
FEATURES OF PARANEOPLASTIC SYNDROME
Hematological Polycythemia
Endocrine Hypoglycemia

TUMOR RUPTURE (<3%)


Presentation Severe abdominal pain (peritonism)
Pallor with shock
Ix US = positive for peritoneal fluid
FAST = positive for blood

UiTM 79
TEMPLATE CASE BASED SCENARIO EXAMINATION (CBSE) 20/21

DIFFERENTIAL DIAGNOSIS RISK FACTORS


ALCOHOLIC CIRRHOSIS
ASCENDING CHOLANGITIS alcohol abuse

NON ALCOHOLIC CIRRHOSIS


CHOLANGIOCARCINOMA Non alcoholic Non – alcoholic fatty liver disease
steatohepatitis
(NASH)
HEPATOMA (HEPATOCELLULAR ADENOMA) Infection Chronic Hepatitis B
Chronic Hepatitis C
Autoimmune Primary Biliary Cirrhosis (PBC)
Secondary Biliary Cirrhosis
COMPLICATIONS FROM THE DISEASE  Chronic obstruction of
COMPLICATION FROM THE TREATMENT the biliary tree
(secondary to PSC) –
usually cause
cholangiocarcinoma but
can also cause HCC
 Recurrent infection in
the biliary tree –
recurrent pyogenic
cholangitis
 Recurrent stricture
formation (iatrogenic
injury) – lead to
secondary BC

Metabolic Hemochromatosis
Wilson’s Disease
Alpha 1 AntitrypsinDeficiency

Others 1. Diet – red meat &


saturated fats
2. Aflatoxins B1 (mouldy
peanuts / corn /
soybeans) – linked with
p53 mutation
3. Diabetes – independent
RF for HCC95
4. Smoking
5. Alcohol – synergistic
effect in individuals with
chronic HBV/HCV

UiTM 80
TEMPLATE CASE BASED SCENARIO EXAMINATION (CBSE) 20/21

PHYSICAL EXAMINATION
MUST GET GOOD TO GET
MAIN CLINICAL CONSEQUENCES OF PORTAL
HYPERTENSION
4. Ascites
5. Formation of portosystemic shunt
6. Splenomegaly

- Per rectal : Melena


Reduced serum Leuconychia
albumin Ascites
Impaired  Palmar erythema
estrogen  Loss of secondary sexual
metabolism characteristic (axillary and
pubic hair loss, testicular
atrophy)
 Gynaecomastia
 Spider naevi

Portal Splenomegaly
GENERAL
hypertension Caput medusa
4. Oriented to time, place and person
5. Vital sign : BP, RR, PR, temperature Hepatic Not oriented to TPP
(Hemodynamic status) encephalopathy Flapping tremor
6. Body built (cachexic)
HAND 24. Clubbing
25. Leuconychia
26. Palmar erythema
27. Flapping tremor
28. Dupuytren contracture
ARM 29. Loss of axillary hair
30. Bruising
31. Needle mark
32. Scratch mark
33. Tatoo
EYE 34. Conjunctival pallor
35. Sclera jaundice
MOUTH 36. Glossitis
NECK, FACE, 37. Lymphadenopathy
CHEST 38. Gynaecomastia
39. Spider naevi
ABDOMINAL 40. Caput medusa
41. Abdominal distension
(ascites) - umbilicus (normal,
flat, everted)
42. Shrunken liver
43. Hepatospenomegaly
INGUINAL 44. Hernia (ascites)
45. Testicular atrophy
LEG 46. Bilateral pitting edema
.

UiTM 81
TEMPLATE CASE BASED SCENARIO EXAMINATION (CBSE) 20/21

PART 2

CASE SCENARIO
Name -
Age 60 years old
Race, Gender Indian
Underlying Hepatitis B for 3 years
Chief complaint RUQ abdominal pain
Duration 3 months prior to the admission
Associated symptoms (if any) + duration Vomiting & weight loss

FULL DIAGNOSIS
Case summary Mr Af, 65y/o gentleman with underlying of hepatitis B for 3 years presented
to Hospital with right sided cramping abdominal pain associated with
persistent vomiting and weight loss of 6 kgs in 3 months.

Нe vomiting was intermittent once in 2-3 days in the beginning, but


progressively increased to 2-3 times daily in the last one month .

On examination patient was dehydrated, sclera jaundice was noted and there
was fullness in the upper abdomen with visible peristalsis and tender
hepatomegaly 3 cms, below the right coastal margin
Presentation Hepatocellular carcinoma
Complicated with Features of chronic liver disease

UiTM 82
TEMPLATE CASE BASED SCENARIO EXAMINATION (CBSE) 20/21

INVESTIGATION
MUST GET GOOD TO GET

DIAGNOSTIC ALPHA FETOPROTEIN (AFP)


1. Triphasic or 4 phase CT scan  When used as a diagnostic test, AFP levels (20
2. Contrast enhanced MRI ng/ml) show good sensitivity but low
3. AFP specificity
4. Ultrasound  A rise in serum AFP in a patient with cirrhosis
5. Biopsy should raise concern that HCC has developed

TRIPHASIC @ 4 PHASE CT SCAN False negative


1. Pregnancy
2. Cirrhosis
3. Hepatitis
4. Teratoma
5. Gastric cancer

ULTRASOUND
Accurate in detection of HCC especially when coupled
with concomitant AFP elevation

BIOPSY / HISTOLOGY
If required, laparoscopic or image guided
percutaneous biopsies can be used to obtain tissue
diagnosis – pathological hallmark (HCC): stromal
HCC  Arterial hypervascularity invasion
 Venous/late phase washout
Additional CT scan also use for Tissue diagnosis is not required before therapeutic
 LN involvement intervention if other modalities favour HCC as the
 Metastases to the adrenals diagnosis

UiTM 83
TEMPLATE CASE BASED SCENARIO EXAMINATION (CBSE) 20/21

STAGING & PROGNOSTIC TO ASSESS COMPLICATION

LIVER FUNCTION TEST

CHILD PUGH CLASSIFICATION of CLD / CIRRHOSIS


1. Prognosis – as it helps to quantitate residual liver function
2. Strength of medical treatment
3. Necessity of liver transplant

UiTM 84
TEMPLATE CASE BASED SCENARIO EXAMINATION (CBSE) 20/21

MANAGEMENT
MUST GET GOOD TO GET

DEFINITIVE : CURATIVE
Surgical With RFA (radiofrequency ablation)
resection
 the treatment of choice for non-cirrhotic patients with HCC
 Only about 10-20% of patients with HCC will have disease amenable to surgery

Poor prognostic factors


1. Micro vascular invasion
2. Multi-nodular tumours
Liver CRITERIA
transplant 1. MELD score
2. Milan
3. UCSF (University of California, San Francisco)
4. Barcelona Clinic Liver Cancer group

In hepatitis B carriers, there is a risk for reinfection of the donor liver (high risk factors are
HBeAg positivity, high HBV DNA levels) – can be aggressively treated with anti-viral drugs 2
months before transplant and anti-HBV immunoglobulin long-term after transplant

Problems with availability of donor organ – the disease might have progressed past being
suitable for transplant by the time donor organ is available

Possibility of “bridging therapy” such as RFA, TACE, Yttrium-90 to shrink disease and prevent
progression until donor liver is available

UiTM 85
TEMPLATE CASE BASED SCENARIO EXAMINATION (CBSE) 20/21

PALLIATIVE

LOCAL
Radiofrequency Best result for loco – regional strategies
ablation (RFA)
Option for patients with early stage HCC not suitable for resection or transplant
Can also be for down staging tumours with subsequent liver transplant
A needle electrode is placed in the tumour, destroying tissue by heating it to temp of 60
to 100 deg.

Percutaneous Microwave / Thermal Ablation Cryotherapy / Cryoablation


ethanol injection
(PEI)

REGIONAL
Trans – arterial MOA
chemoembolization Selective intra-arterial administration of chemotherapeutic agents
(TACE) +
followed by embolization of major tumour artery

Contraindications
1. Portal thrombosis
2. Both blood supply decrease leading to worsening liver function

Complications
1. Fever (secondary to cytokine release as a result of tumour lysis – TNF alpha,
IL-1), abdominal pain, nausea, vomiting
2. LFTs – deranged raised ALT and AST (reflection of ischemic hepatitis) –
hepatic failure due to infarction of adjacent normal liver (TACE should not
be used for Child’s class C cirrhosis)
Trans-arterial
embolization (TAE)
Selective Intrahepatic Yttrium-90 radioactive beads injected into hepatic artery  irradiating the tumour
Radiation

SYSTEMIC
Sorafenib Multi-kinase inhibitor
200mg BD Have anti-angiogenic and anti-proliferative properties

Sorafenib used in combination with TACE in advanced primary HCC – superior to Sorafenib
monotherapy – ?impact on overall survival

UiTM 86
TEMPLATE CASE BASED SCENARIO EXAMINATION (CBSE) 20/21

OBSTRUCTIVE JAUNDICE

CASE SCENARIO
Name
Age
Race, Gender
Underlying
Chief complaint Yellowish discolouration of skin and eye
Duration
Associated symptoms (if any) + duration Abdominal pain

Presentations of cholelithiasis:
1. Asymptomatic
2. Symptomatic- Biliary colic
3. With complications

PART 1

HISTORY
MUST GET GOOD TO GET
EXPLORE THE CHIEF COMPLAINT
Biliary colic
Obstructive jaundice RUQ pain
1. Is it jaundice? S RUQ/Epigastric
2. Direct or indirect? O Within hours of eating meal, often awakening
3. Pre/Hepatic/Post hepatic patient from sleep
C Waxing-waning, rarely have pain-free intervals
Yellowish discolouration of skin and eye (Not a true colic)
S Sclera R Inferior angle of right scapula
Skin- Palm/Sole/Feet Tip of right shoulder
O Sudden  Stones A 1. LUQ pain
Gradual  Malignancy 2. Back pain
C - 3. N&V (better after vomiting)
R -- 4. Bloating
A Painful  Stone, Stricture, Hepatic causes 5. Abdominal distension
Painless  Malignancy (HOP tumor, T Periodicity:
cholagioCA) Pain comes in distinct attacks lasting 30mins-
T Intermittent  Ascending cholangitis several hours
Episodic  Choledocholithiasis E Resolves spontaneously
Progressive  Malignancy S Steady and intense
E -
S -

UiTM 87
TEMPLATE CASE BASED SCENARIO EXAMINATION (CBSE) 20/21

ASSOCIATED SYMPTOMS DIFFERENTIAL DIAGNOSIS


1. If painful 
Choledo- Epigastric/RUQ pain Painful jaundice
cholithiasis Pain more prolonged than Choledocholithiasis
biliary colic Ascending
Cholecystitis RHC pain cholangitis
Constant, unremitting, severe Post-
pain (d/t inflammation instrumentation
spreading to parietal strictures
peritoneum) Acute cholecystitis
Radiates to inferior angle of Hepatic causes
scapula and interscapular *if pain w/out jaundice, don’t forget TRO PUD, renal
Ascending RHC pain colic, GERD, inferior MI, lower lobe pneumonia
cholangitis Fever
Confusion Painless jaundice
Gallstone Abdominal pain radiating to the CholangioCA
pancreatitis back Periampullary tumor
N&V

2. If painless 
Constitutional LOA, LOW, Lethargy COMPLICATIONS FROM THE DISEASE
symptoms
Metastatic Bone pain, dyspnea, neck lump Liver Stigmata of CLD
symptoms decompensation Ascites
Pain Constant Bleeding tendency
Late symptoms of pancreatic CA Confusion
Fat malabsorption Steatorrhea
3. Direct or indirect?
Direct/ Tea coloured urine
Conjugated Pale stool
Pruritus
Indirect Normal urine and stool

4. Pre hepatic/Hepatic/Post hepatic causes


Pre Jaundice
Pallor
Hepatic Any stigmata CLD

Viral hepatitis
Prodromal fever
Malaise
Arthralgia
Myalgia
N&V

HCC
Abdominal distension
Post Obstructive jaundice

5. Others
- N&V

UiTM 88
TEMPLATE CASE BASED SCENARIO EXAMINATION (CBSE) 20/21

RISK FACTORS

Pre Personal/Family history of bleeding


disorder/blood transfusion
Hepatic History of alcohol abuse
Travel history
Ingestion of seafood
Family history of hepatitis
Blood transfusion
Needle stick injuries
Drug abuse/Needle sharing
Sexual contact
Post Parasitic infection

Past history of gallstone disease/


biliary colic/ surgery/ ERCP
Forty
Female
Fat
Fertile

Elderly
Family history of malignancy

UiTM 89
TEMPLATE CASE BASED SCENARIO EXAMINATION (CBSE) 20/21

PHYSICAL EXAMINATION
MUST GET GOOD TO GET

General Metastasis
Obesity Gallstone Cervical, supraclavicular Lymphadenopathy
Cachexia Malignancy LN
Jaundice Sclera, Skin Bony tenderness
Stigmata CLD CLD Respiratory examination Pleural effusion
If on CBD Tea coloured urine

Vitals
Hemodynamically Shock
stable/ unstable
Febrile Infection

Abdomen
Abdominal distension CLD
Generalized distension Malnutrition
Peritoneal malignancy
Obstruction of portal
vein by cancer
Scars
Hepatomegaly Metastatic dz
HCC
Enlarged palpable Courvoisier’s law (Cause
gallbladder other than stone)
Splenomegaly Portal HPT
DRE Pale stool

PART 2

FULL DIAGNOSIS
Presentation Painless/Painful obstructive jaundice
Secondary to
With underlying (if any)

UiTM 90
TEMPLATE CASE BASED SCENARIO EXAMINATION (CBSE) 20/21

INVESTIGATION
MUST GET GOOD TO GET
LABORATORY IMAGING

Confirm obstructive jaundice Gallstones


LFT High bilirubin (direct>indirect, Ultrasound Gallstone/Cholecystitis/
Normal 3:7 HBS Choledocholithiasis
High ALP - Duct dilation >6mm
High GGT Best - GB stones or sludge
- only ordered if diagnosis is imaging for - Thickening GB wall
ambiguous, not routine gallstones - Pericholecystic fluid
Mildly raised liver enzymes ie ALT, - Fat stranding
AST - Complications ?
- Biliary backpressure  Liver consistency
hepatocyte damage - Fatty or cirrhotic
Disadvantages:
Underlying inflammation/infection 1. Unable to detect
FBC Leukocytosis malignancy
- Suggest bacterial infection, 2. Unable to detect distal CBD
important as some patient stones
have not mounted febrile 3. Sensitivity reduced with fat
response patient habitus
Anaemia 4. Operator dependent
- Correct before ERCP Plain Only detect 10% gallstones due to
Thrombocytopenia abdominal radiolucent
- CLD, Viral infection X-Ray
CT scan Complications of gallstones
Assess potential complication - Perforated GB: Air around GB
RP Dehydration Usually not in lung window
Hepatorenal syndrome done to Rule out malignant etiology
Creatinine level for CT imaging diagnose
stones
Serum Any concomitant pancreatitis
amylase
CP Coagulopathy
- VitK malabsorption/liver
dysfunction
- Need to be corrected before
procedures
Blood C&S If febrile and jaundiced
- TRO HBS sepsis
Tumor CA 19-9  Pancreatic CA
markers CEA  CholangioCA
GXM If need operation

UiTM 91
TEMPLATE CASE BASED SCENARIO EXAMINATION (CBSE) 20/21

Cholangiocarcinoma
Ultrasound HBS 1. Biliary duct dilatation
2. Localize site of
obstruction
Hilar Intrahepatic
lesions duct dilatation
with normal
extrahepatic
ducts
Distal Intra and
lesions extrahepatic
ducts dilated
Contrast- Intrahepatic cholagioCA
enhanced triple
phase, helical CT
Cholangiography Non-invasive = MRCP
- Superior to ERCP for
assessing tumor
anatomy and
resectability
Invasive = PTC/ERCP
- Cytologic analysis via
ductal brushing or FNA
Endoscopic U/S Cytologic diagnosis and assess
with FNA nodal involvement
PET-CT
CT TAP Metastatic spread

Endoscopic -
Retrograde -
Cholangio-
pancreatography -
(ERCP)
Percutaneous Assess intrahepatic ductal
Transhepatic disease
Cholangiography - Sclerosing cholangitis
(PTC) - CholangioCA
Obtain brushing and biopsies
to evaluate for malignancy

UiTM 92
TEMPLATE CASE BASED SCENARIO EXAMINATION (CBSE) 20/21

MANAGEMENT
MUST GET
ACUTE

Resuscitation
“In view of ascending cholangitis being a surgical emergency, I would like to resuscitate the patient who may
be in septic shock.”

Call for help/Inform


seniors
Obtain IV access
IV fluid resuscitation Strictly monitor input and output
Take blood for Most important blood C&S
investigations
Close monitoring of HR, RR, BP
vitals Keep MAP >65 mmHg
Inform doctors if SBP <100mmHg
CVP line insertion if patient unresponsive to fluid resuscitation
Keep 10-12 mmHg
Insert CBD + monitor Maintain >0.5 mg/kg/hr
urine output

Treat the infection


IV antibiotics IV Ceftriaxone and Metronidazole
Imipenem if in shock

Treat the obstruction = Emergent biliary decompression

- Usually deferred until 24-48 hours after admission when patient is stable or has improved with
systemic antibiotics.
- Emergency if deteriorating or infection not improved with antibiotics

ERCP Diagnostic and therapeutic role


(commonly used) Procedures
- Correct anaemia and coagulopathy prior. Stop anticoagulants 5 days
prior
- Performed at outpatient setting or inpatient (emergency)
- IV sedation and analgesics (local anaesthesia spray in throat)
- Antibiotic prophylaxis- suspect biliary obstruction/ known pancreatic
pseudocyst/ ductal leak

Biliary Choledocholithiasis (mainly therapeutic)


disease  Therapeutic : sphincterotomy and stone extraction
(balloon / wire basket KIV stent)
 Pre-operative : emergent ERCP if worsening cholangitis
 Post-operative : elective ERCP to remove retained CBD
stones

UiTM 93
TEMPLATE CASE BASED SCENARIO EXAMINATION (CBSE) 20/21

Malignant and Benign Biliary Strictures


 Diagnostic : brushing, biopsies & FNAC (combined
sensitivity 62%)
 Therapeutic : stricture dilatation (hydrostatic balloon /
graduated catheter) KIV stenting

Post-operative Biliary Leaks


 Therapeutic : endoscopic stenting to decompress bile
ducts – Sphincter of Oddi Dysfunction
 Therapeutic : endoscopic sphincterotomy

Pancreatic Recurrent Acute Pancreatitis


disease  Diagnostic: obtain definitive imaging of ductal anatomy
 Therapeutic: endoscopic sphincterotomy

Chronic Pancreatitis (and its associated complications – i.e.


strictures)

Pancreatic Duct Leaks


 Therapeutic: pancreatic-duct stent

Pancreatic Fluid Collection – i.e. pseudocysts, pancreatic necrosis


 Therapeutic: pancreatic stent & sphincterotomy – drain
pseudocyst that communications with pancreatic duct

Pancreatic Malignancies
 Diagnostic: ERCP directed brush biopsy / FNAC (combined
sensitivity 65%)

Ampullary Ampullary Adenoma


disease  Therapeutic: snare ampullectomy with combined biliary
and pancreatic sphincterotomy

Complications
1 Pancreatitis – 3-4%  Irritation of the pancreas duct
(usually settles by by the injection of contrast
itself)  Edema when removing stone
2 Infections 1-2%  Cholecystitis
 Cholangitis
3 Haemorrhage 1-2% Result from cutting the opening of
the duct (sphincterotomy)
4 Perforation 0.5% Duodenal or biliary
5 CVS and/or 1-2% Risk of sedation:
analgesia-related  ↓ BP
 ↓ O2 levels and respiratory
depression
6 Fatality 0.33%
.

UiTM 94
TEMPLATE CASE BASED SCENARIO EXAMINATION (CBSE) 20/21

Percutaneous Involves transhepatic needle insertion into a bile duct  Injection of contract to
transhepatic opacify the bile ducts
cholangiography
(PTC)/ Indications
Percutaneous - Patients who have biliary duct dilatation on ultrasound or other
transhepatic biliary imaging modalities and are not candidates for ERCP
drainage (PTBD) Diagnostic
1. Assess intrahepatic ductal disease (i.e. sclerosing cholangitis &
cholangiocarcinoma)
2. Obtain brushing, biopsies to evaluate for malignancy
Therapeutic interventions
1. Drainage of infected bile in cholangitis
2. Extraction of biliary tract stones
3. Dilatation of benign biliary strictures
4. Placement of stent across a malignant stricture
Complication (for most transhepatic procedure)
1. Cholangitis ~2%
2. Hemobilia – communication between biliary duct and vascular structure
 Hepatic artery pseudoaneurysm
 Hepatic artery-bile duct / portal vein fistula

DEFINITIVE

Gallstones Laparoscopic cholecystectomy

 Hasson Open Technique – pneumoperitoneum established


 GB dissected from GB bed and removed via umbilicus with the use of an
applied medical bag
 GB bed is irrigated and haemostasis checked
 Recovery – 80% of patients discharged within 24 hours, most discharged
by POD2

Open cholecystectomy

- Indications
1. Obesity
2. Previous cholecystitis
3. Prior upper abdominal surgery (adhesions ++)
- Based on patient safety (i.e. safe dissection can’t be performed
laparoscopically) and not considered a complication of surgery

UiTM 95
TEMPLATE CASE BASED SCENARIO EXAMINATION (CBSE) 20/21

Complications

Specific to 1. Injury to the bile duct – 0.25%


Procedure 2. Injury to neighbouring structures (i.e. hepatic
artery and bowel)
3. Un-retrieved gallstone spillage – LT risk of abscess
and fistula formation
4. Retained stones in the CBD – requires subsequent
management – <5%
5. Hernia at incision site – esp. in obese patients
6. ? post-cholecystectomy syndrome

Specific to General 1. Allergic reaction to anaesthetic


Anaesthesia 2. Lung infection, Stroke, heart attack, death

Related to
laparoscopic
surgery
Related to any Bleeding, wound infection, DVT, PE
surgery
.
+/- CBD exploration

Cholangiogram or Cholangiogram
choledochoscopy is - Injection of dye  can image higher
performed ducts
Choledochoscopy
- Using scope to visualize large biliary
ducts  cannot image higher ducts, thus
not sensitive
- But can used to remove stones
visualized in the duct

*Choice of imaging depends on site and cause of


obstruction
Removal of stones 1. Manual removal with stone-grasping forceps
2. Flushing out stones
3. Dredging stones out using balloon catheter
or Dormia basket
4. Lithotripsy
.

Lot of instrumentation of biliary system being used during operation 


Swelling and edema of biliary system 
Post-operative obstruction and build-up of bile 
Higher risk of biliary leakage

Stent Removed later by endoscopy


T-tube - T-shaped tube with horizontal limb placed in CBD and
(usually vertical limb leading out to drain bile
inserted after - Function: “Pressure release valve”

UiTM 96
TEMPLATE CASE BASED SCENARIO EXAMINATION (CBSE) 20/21

CBD - Allows for post-op cholangiogram to check for remaining


exploration) stones before removal: Free flow of contrast into
duodenum, no residual stones, no free leakage of
contrast into peritoneum
- If all normal  Release anchoring stitch and exert gentle
traction on the tube. The tube should slip out easily, if
not call for help.
- If stones are present  Leave tube in for 4-6 weeks to
form a fibrous tract  Allows for instrumentation of tract
with a scope to remove stones
Cholangiocarcinoma Complete surgical resection with histologically negative margins

Extrahepatic cholangioCA affecting Resection of biliary tree and hilar


CBD lymphatics
Hilar (Klatskin’s) tumor Above + partial hepatectomy
Distal cholangioCA Pancreatoduodenectomy (Whipple’s
procedure)

UiTM 97
TEMPLATE CASE BASED SCENARIO EXAMINATION (CBSE) 20/21

Painful Hematuria: Urolithiasis

CASE SCENARIO
Name -
Age 55 years old
Race, Gender Men
Underlying -
Chief complaint Hematuria with loin to groin pain
Duration 2 weeks
Associated symptoms (if any) + duration

PART 1

HISTORY
MUST GET GOOD TO GET
EXPLORE THE CHIEF COMPLAINT DIFFERENTIAL DIAGNOSIS (HEMATURIA)
Stones Nephrolithiasis (kidney stones)
CLINICAL PRESENTATION OF STONES Ureterolithiasis (ureter stones)
1. Obstruction Cystolithiasis (bladder stones)
2. Hematuria (due to ulceration)
3. Chronic infection Infections 1. Urinary Tract Infection
2. Cystitis (bladder)
Hematuria 3. Prostatitis (prostate)
O Onset 4. Tuberculosis
D Duration
F How many episode Trauma 1. Urinary Catheterisation
A Painful = infection, inflammation 2. Flexible Cystoscopy
Painless = malignancy 3. Post-TURP
T When during urination does the blood
appear? Tumor  Transitional Cell Carcinoma
Initial Urethra (bladder, ureter)
Terminal Near bladder neck or  Prostate Cancer / Benign
prostatic urethra Prostate Hyperplasia (BPH)
Throughout Upper urinary tract
Bladder
RULE OUT BENIGN CAUSES OF HEMATURIA
O  Presence of blood clot 1. Menstruation
(extraglomerular bleeding) 2. Exercise – induced myoglobinuria (vigorous
 Frothy urine (glomerular bleeding) exercise)
3. Sexual intercourse
Loin to groin pain (SOCRATES) 4. Trauma - Any procedure involving urinary
C Intermittent colicky pain tract (catheter, cystoscopy)

UiTM 98
TEMPLATE CASE BASED SCENARIO EXAMINATION (CBSE) 20/21

ASSOCIATED SYMPTOMS RISK FACTORS


Urinary 1. Any frothy urine MODIFIABLE
2. Storage symptoms : 1. Diet
frequency, urgency, nocturia 2. Dehydration – low urine volume
3. Voiding symptoms : 3. Hypervitaminosis D – massive ingestion of
dribbling, intermittent vitamin D
stream, poor stream, 4. Milk-Alkali Syndrome – hyperCa2+ due to
hesitancy repeated ingestion of Ca2+ & absorbable
4. Dysuria alkali
5. Loin mass

Anemic
Coagulopathy NON – MODIFIABLE
Constitutional LOA, LOW 1. Age (majority occur during the 4th – 6th
Infection Fever decade of life)
Nausea and vomiting 2. Gender (M:F – 3:1)
Recent throat or skin infection 3. Cystinuria – inherited AR disease
Metastatic SOB, jaundice, bone pain 4. Inborn error of purine metabolism
Paraneoplastic Symptoms of hypercalcemia 5. Chemotherapy – excess uric acid following
syndrome 1. Loss of appetite. treatment of leukaemia / polycythaemia
6. Idiopathic hypercalciuria
2. Nausea and vomiting.
7. Gout
3. Constipation and
abdominal (belly) pain.
STONE FORMATION
4. The need to drink more
fluids and urinate more. SYSTEMIC DISEASE / METABOLIC DISORDER
5. Tiredness, weakness, or 1. Crohn’s disease
muscle pain. 2. Hyperparathyroidism, hyperthyroidism
6. Confusion, 3. Gout
disorientation, and 4. Renal tubular acidosis
difficulty thinking. 5. Metastatic cancer
7. Headaches. 6. Paraneoplastic syndrome
8. Depression
FAMILY, DRUG, SOCIAL HISTORY
1. Family history of stones
2. Medication (antacids, salicylic acid,
acyclovir)
3. Occupation history
4. Diet : high protein and sodium intake

COMPLICATIONS FROM THE DISEASE


COMPLICATION FROM THE TREATMENT

UiTM 99
TEMPLATE CASE BASED SCENARIO EXAMINATION (CBSE) 20/21

PHYSICAL EXAMINATION
MUST GET GOOD TO GET
 In ureteric colic, symptoms are often out of POINTS OF CONSTRICTION
proportion to signs – no guarding, rebound 1. Pelvic-ureteric junction (PUJ)
 If the patient has pyelonephritis, renal punch 2. Pelvic brim (near bifurcation of the common
may be positive iliac arteries)
 Otherwise unremarkable examination 3. Veisco-ureteric junction (VUJ) – entry to the
bladder
GENERAL
Vital signs fever (pyelonephritis)
HTN (glomerulonephritis)
Anemia Conjunctival pallor
Palmar crease pallor
Extremities Edema (glomerulonephritis)
Rashes (HSP, CTD, SLE)

SYSTEMIC
Heart – new murmur (endocarditis)
Lungs – crackles, rhonchi (Goodpasture’s syndrome)

Abdominal
a. Loin tenderness, renal mass, bruit (renal
ischemia), palpable bladder, suprapubic mass
b. Are the kidneys ballotable, is the bladder
percussible?

c. Scrotum – varicocele on the left (may have


RCC of the left kidney with extension of
tumour into renal vein, blocking the testicular
vein where it drains into the left renal vein)

d. External genitalia – blood from urethra, any


blood clots, shape of blood clot

e. Digital rectal exam – prostate enlargement


(BPH versus cancer)

PART 2

CASE SCENARIO
Name -
Age 55 years old
Race, Gender Men
Underlying -
Chief complaint Hematuria with loin to groin pain
Duration 2 weeks
Associated symptoms (if any) + duration

UiTM 100
TEMPLATE CASE BASED SCENARIO EXAMINATION (CBSE) 20/21

FULL DIAGNOSIS
Presentation
Secondary to Ureterolithiasis
With underlying (if any)

INVESTIGATION
MUST GET GOOD TO GET
OUTLINE TO ASSESS COMPLICATION
1. Blood investigation
2. Urine test TYPES OF CALCULI
3. CT scan 1. Calcium oxalate (75%)
4. KUB X – ray 2. Struvite stones (15%)
5. Intravenous urogram (IVU) 3. Urate stones (5%)
6. US of kidney & bladder 4. Cystine stones (2%)
7. MAG – 3 renogram 5. Xanthine (rare)
6. Pyruvate stones (rare)
BLOOD INVESTIGATION
FBC Anemia CT SCAN
Leukocytosis (mild)
Serum Calcium If raised do PTH CT KUB for first time presentation
LFT Albumin (non-
Serum uric acid contrast) Replaced IVU as the diagnostic test
of choice in the acute setting to
evaluate for stones
URINE TEST
24 hour urine Calcium
CT Urogram 1. No contrast – any stones,
collection of Sodium
(tri-phasic) gross abnormalities
metabolic Phosphate
2. Medullary – any cysts,
profile Magnesium
parenchyma abnormalities
Oxylate
3. Delayed phase – any filling
Uric acid
defects
4. Evaluate anatomy and
Haematuria Gross
reflects renal function
Microscopic
UTI Pyuria
Microorganisms
ULTRASOUND KUB
Ph of urine Acidic vs Alkaline stones
 Any evidence of kidney stones or
complication of stones – i.e. hydronephrosis
INTRAVENOUS UROGRAM (IVU)
 Choice for patients with contrast allergies and
 Help visualise uric acid stone (detects
pregnant females
radiolucent stones)
 Features of stone: echogeneic rim, posterior
 Shows dilated urinary system 20 to stone
obstruction – hydroureter and/or acoustic shadowing
hydronephrosis
 Rough indication of renal function

UiTM 101
TEMPLATE CASE BASED SCENARIO EXAMINATION (CBSE) 20/21

MANAGEMENT
MUST GET GOOD TO GET
OUTLINE CONSERVATIVE
Pain control Analgesia 1. High fluid intake
IM Pethidine 2. Diet modifications
Tramadol 3. Chemical dissolution
Infection Antibiotics 4. Others : Narcotic pain medications + daily
(UTI) alpha blocker (Tamsulosin)
Stones Allow for spontaneous passage of
stones or decide on active stone Duration
removal  can be used for 4 – 6 weeks
Underlying  Bladder stones sec to BPH tx  stop if there is a symptoms
aetiology with TURP
 Hypercalcaemia  treat
disease if possible

TREATMENT MODALITIES
LOCATION SIZE (mm) TREATMENT
Renal <5 Conservative management unless symptomatic / persistent
5 – 10 ESWL
10 – 20 ESWL or PCNL
>20 PCNL
Upper ureter <5 Conservative management unless symptomatic / persistent
5 – 10 ESWL
>10 URS with lithotripsy
Middle / distal <5 Conservative management unless symptomatic / persistent
ureter >5 URS with lithotripsy
ESWL
Bladder <30 Cystolitholapaxy
>30 Open cystolithotomy (also if there are multiple stones)
.
SURGICAL INTERVENTION TYPES
INDICATIONS ESWL Extracorporeal Shock Wave Lithotripsy
7S Size, site, symptoms, stasis, stuck, PCNL Percutaneous Nephrolithotomy
sepsis, social URS Ureteroscopy lithotripsy
S/S Constant pain
Stone  Obstructs urine flow
complications  Causes urinary tract infection COMPLICATIONS OF TREATMENT
 Damages renal tissue or 1. Hematoma / Significant Bleeding
causes significant bleeding 2. Urinary tract infection
 Increase in size 3. Ureteric Injury – perforation / ureteric
Unlike to  Does not pass after one avulsion
resolve with month 4. Failure of procedure –i.e. unable to assess
conservative  Too large to pass stone
treatment spontaneously
.

UiTM 102
TEMPLATE CASE BASED SCENARIO EXAMINATION (CBSE) 20/21

LUTS: BPH/ PROSTATE CANCER

CASE SCENARIO
A 68-year old man presents with inability to pass urine associated with suprapubic pain since early morning.

PART 1

HISTORY
MUST GET GOOD TO GET
EXPLORE THE CHIEF COMPLAINT PMH
1. Onset, when pass urine last time  Previous catheterization
2. Seek treatment
3. What did the doctor do?  Surgical
4. First/repeated episode?  Suffer from any injury or undergone a
5. Amount of water - Alcohol/beer surgical operation in the
groin/perineum
ASSOCIATED SYMPTOMS
-LUTS-  Drug
Storage Voiding  Drug can cause urinary retention
1. Frequency 1. Hesitancy Anti depressant : weaken bladder
2. Nocturia 2. Poor flow contraction
3. Urgency 3. Intermittent stream  Increase internal uretheral sphincter
4. Urge 4. Dribbling
incontinence 5. Sensation of poor  Family history
5. Enuresis bladder-emptying  Prostate Ca
(nocturnal 6. Episodes of near
incontinence) retention  Social history
- Hematuria  Smoking
 Alcohol
 Constitutional symptom
 Bone pain COMPLICATIONS
 LOA 1. Chronic urinary stasis
 LOW  Recurrent UTI - frequency, dysuria
 Rule out neurogenic causes  Stones - hematuria
 Renal impairment

RISK FACTORS
1. Age : >50 years old
2. Genetic and racial : Caucasian higher than Asian
group
3. Diet : High protein diet

UiTM 103
TEMPLATE CASE BASED SCENARIO EXAMINATION (CBSE) 20/21

PHYSICAL EXAMINATION
MUST GET GOOD TO GET
ABDOMINAL EXAMINATION Other Examination
 Palpate spine for any bone pain, pathological #
INSPECTION Abdominal distension  Ballot Kidneys – any hydronephrosis
PALPATION Ballotable kidney  Examine for lymphedema

DRE:
BPH Lateral enlargement
Prostate Ca Asymmetrical, hard, irregular,
craggy enlargement of prostate
palpable PR
Sensation of perineal skin and tone of anal sphincter
to rule out neurogenic cause

DIFFERENTIAL DIAGNOSIS
Prostate Cancer  Hematuria
 LOA/LOW
 Bone pain
Uretheral  LUTS
stricture  Risk Factor
 A medical procedure that involves inserting an instrument, such as an
endoscope, into the urethra
 Intermittent or long-term use of a tube inserted through the urethra to drain
the bladder (catheter)
 Trauma or injury to the urethra or pelvis

UiTM 104
TEMPLATE CASE BASED SCENARIO EXAMINATION (CBSE) 20/21

PART 2

FULL DIAGNOSIS
Presentation Acute Urinary Retention
Secondary to Benign Prostate Hyperplasia
With underlying (if any)

INVESTIGATION
MUST GET GOOD TO GET
DIAGNOSTIC Staging
1. Full blood count - Infection (Urinary tract 1. Clinical examination (palpable tumour : T2)
infection (UTI) 2. TRUS biopsy for staging purpose
2. Urinalysis - Urinary tract infection (UTI) a) Procedure-related complications – risk of
3. Urine culture – Urinary tract infection (UTI) sedation, bleeding (PR bleed,
4. Serum creatinine – Renal impairment haematochezia, hematospermia), infection,
5. Uroflowmetry - Urinary flow rate and post- urosepsis (1% chance of serious infection
voiding residual volume that require hospital stay – give prophylactic
6. Prostate Specific Antigen – BPH, prostate cancer antibiotics (gentamicin)
 10ng/ml: biopsy recommended as 67% 3. CT scan of the abdomen and pelvis to assess
of patients will have prostate cancer extent of tumour invasion and nodal status
 4-10ng/ml: biopsy advised, though (regional, non-regional)
only 20% will have prostate cancer 4. Bone scan for metastasis – if PSA < 20 chance of
 < 4ng/ml: majority will have negative Mets = 5%
biopsies, and yet there is a significant
proportion of men with prostate
cancer with PSA <4ng/ml
7. Cystourethroscopy – urethral stricture, bladder
carcinoma, bladder calculi
8. Ultrasound KUB - Hydronephrosis
9. Transrectal ultrasound scanning – not done
routinely

UiTM 105
TEMPLATE CASE BASED SCENARIO EXAMINATION (CBSE) 20/21

MANAGEMENT
MUST GET GOOD TO GET
DEFINITIVE CONSERVATIVE (Lifestyle, diet etc)
Conservative (watchful waiting) – (IPSS 8, insignificant
PHARMACOLOGY PRV residual)
IPSS 8-18 moderate sx, residual <100mls  Bladder training
 Fluid modification
 a-blockers (Prazosin, Alfuzosin, Tamsulosin,  Avoiding/ monitoring certain drugs: Diuretics
Terazosin)  Avoiding Constipation
 MOA : inhibit the contraction of  Symptom monitoring
smooth muscle in the prostate  Assess quality of life – IPSS
 Relieve symptoms within a few weeks  Follow-up
(rapid) but do not stop the process of
prostate enlargement

 5a-reductase inhibitor
 MOA : inhibit the conversion of
testosterone into dihydrotestosterone
(DHT)
 Reduce the size of an enlarged
prostate but may take 6 months or
more to show any effect

 Combination therapy with antimuscarinic


(Duodart)

SURGICAL
 Fail medical therapy
 Chronic retention and renal impairment
 Stone, infection and diverticulum formation
 Haemorrhage (hematuria)
 Severe symptoms which failed medical therapy
1. Transurethral resection of the prostate (TURP)
2. Open prostatectomy

UiTM 106
TEMPLATE CASE BASED SCENARIO EXAMINATION (CBSE) 20/21

MANAGEMENT (PROSTATE CANCER)


MUST GET GOOD TO GET
DEFINITIVE CONSERVATIVE (Lifestyle, diet etc)

1. Radical prostatectomy
Treatment of choice for patients with life expectancy
>10 years
Open, laparoscopic or robot-assisted
Open – retropubic or perineal approaches
Cx: erectile dysfunction, urinary incontinence,
bladder neck stenosis, risks of GA

2. Radiotherapy
External beam radiotherapy (EBRT) – treatments
Interstitial Bradytherapy
Cx: cystitis, prostatitis, bladder over-activity, erectile
dysfunction

LOCALLY ADVANCED DISEASE (T3/4) : Radiotherapy


with androgen ablation

UiTM 107

You might also like